Jennifer Pan

Sir Robert Ho Tung
Professor of Chinese Studies
Professor of Communication
Senior Fellow at FSI
Stanford University

CV

Research

Teaching

Resources

Resources

This section includes resources for social scientists who are learning R, Python, and Latex, as well as summaries of seminal articles and books in comparative politics cover topics ranging from economic development to regime change to party competition.

R and Latex resources were originally prepared as part of Government 2001 and the Harvard University Department of Government Math Prefresher

Summaries were prepared for the spring 2011 General Examination in the Department of Government. Contributors include Ashley Anderson, Iza Ding, Shelby Grossman, Daniel Koss, Jennifer Pan, Evann Smith, Chiara Superti, as well as notes from from the Fall 1998 and Spring 2000 Comparative Politics Field Seminar compilations. Do not cite. Notes on international relations (IR) can be found here.

Shortcuts:

Getting started with Latex

Latex is a document preparation system. To create a document in Latex, a text editor must be used. To get started, you have to download a TeX compiler and a TeX editor.

If you are using Windows, go to MiKTeX, and download the complete version. Double click .exe file to install. If you are using Mac, go here, and double click the MacTeX package to install. This download / installation may take some time, but you MUST wait for the compiler to finish installing before installing the TeX editor.

There are many different text editors, see a comparison here.

Find links below to a basic buide to getting started with Latex, as well as a problem set template for Latex and a Beamer Presentation template.

Getting started with R

R is a programming language for statistical computing and graphics. It’s free, and to get started, download R at cran.r-project.org. If you are using Windows, Start R by double-clicking the program icon. When R launches, a window opens, within the window will be the “R Console”. Go to File and select “New Script”, then the “R Editor” will open. Type and save your code in the Editor, use Ctrl-R as a shortcut to send text from the Editor to run in the Console. When you close R, say “No” to “Save workspace image”; if you select “Yes” you will save the what you have run in the Console during the session, which is unnessary when your worked is saved in the editor. If you are using Mac, Start R by double-clicking the program. When R launches, the Console window will open. Go to File, and select “New Document”, which will open the “R Editor”. Similarly, type code in the Editor, use Command-Enter to send text from the Editor to the Console.

Below is a basic guide to using R and some practice problems for R and Latex

Getting started with Python

Python is a very readable, high-level programming language. For social scientists, many begin using Python in order to collect data, but there are tons of useful libraries such as Numpy and SciPy for scientific computing.

Mac OSX 10.3 and up includes a version of Python, and for Windows users, you can download it from here. Below are some resources to get you started

Modernization

The field of comparative politics was born in the 1960s and 1970s, fueled by questions of modernization–whether the rest of the world would mimic the development of 19th century Western Europe. The summaries below represent classic modernization theory (Deutsch, Inkeles) as well as critiques of modernization theory (Moore, Huntington, Wallerstein, Inglehart and Welzel).

Alex Inkeles, “The Modernization of Man,” in Myron Weiner, ed. Modernization: The Dynamics of Growth (New York: Basic Books, 1966), pp. 138-150.

Inkeles explores the consequences of the process of economic change, or modernization, that was witnessed with the advent of industrialization. Inkeles focuses on the societal consequences of economic modernization and directs our attention to changes in values and cognitive shifts that accompany industrialization. He emphasizes that modernity is not only defined by external conditions such as urbanization and factories but also includes changes within individuals such as mindset and orientation towards technology. Along these lines, Inkeles juxtaposes habit against deliberate choice as the defining characteristics of tradition and modernity, and explores several themes that would signal a shift from “traditional” to “modern man.” The themes that characterisze modern man are: 1) readiness for new experiences and openness to change, 2) disposition to hold opinions over issues that are not immediately related to one’s everyday lifestyle (usually facilitated by education), 3) democratic orientation, including awareness of the legitimacy of diverse attitudes, i.e., not everyone must think alike; as well, does not always accept the ideas of those who are above him or reject the ideas of those below him, 4) oriented to think about the present or future rather than the past and accepts fixed schedules as appropriate and desirable, 5) plans and organizes life, 6) believes that man can learn to dominate his environment to suit his own purposes (efficacy), 7) believes that people around him can be relied on to fulfill their duties, 8) has awareness for the dignity of others and shows respect for them, 9) has faith in technology and science, 10) believes in distributive justice, i.e., people getting as much as they put in. It is important to note that these themes are both consequences of and facilitators of modernity. Economic changes such as the rise of the modern factory make it more likely that individuals will appreciate schedules and technology. But as well, appreciation of schedules and technology make it more likely that factories will be built. In this way, social change is both epiphenomenal and causal in the modernization process, which implies significant feedback between social and economic change. Beyond these themes, it is important to note that Inkeles lists other factors that contribute to modernity. Chief among them is education, because the school serves as a model of rationality and emphasizes the importance of technical competence and distributive justice. However, urbanization, mass communication and the factory are all put fort as important factors in the modernization process.

Karl Deutsch, “Social Mobilization and Political Development,” American Political Science Review, vol. 55 (Sept. 1961)

Like Inkeles, in this piece Deutsch examines the social processes of change that accompany economic mobilization. For Deutsch, the key aspect (consequence) of modernization is the “mobilization” of segments of society in reference to their “availability for recommitment” to new forms of behavior–hence the term social mobilization. In this way, social mobilization implies a two step process: 1) first, old social, economic and technological commitments must be dismantled and 2) individuals must be socialized into new patterns of behaving. Deutsch proposes the following arguments about social mobilization. First, he notes that social mobilization is a coherent variable–that is, the subcomponents of mobilization and interrelated and often accompany one another under certain processes of economic development. Second, he argues that social mobilization is a unilinear process and that its component processes can occur in different countries regardless of their cultural differences. Finally, and most important, he notes that social mobilization processes affect politics. By creating new social forces, social mobilization allows for new groups to have voice or contend for a voice in the political sphere. To sum up his various arguments, Deutsch proposes the following model which elucidates the connection between economic modernization and socio-political change. First, the resultant process of social mobilization that follows economic modernization forms new social strata (for example unionized workers) who demand to be incorporated into politics. Second, these groups then create pressure for the transformation of political institutions such that they can be brought into the political system. Third, as a result of these pressures for change and inclusion, government services are highly demanded and government must adapt by expanding and becoming more efficient. And finally several political consequences result: a bigger bureaucracy is needed, local interests must subordinate to state interests, and the state must turn inward to enhance its functioning. Clearly in this argument the causal arrow goes from economic mobilization processes to sociopolitical mobilization processes. Following his explication of his theoretical model, Deutsch attempts to add empirical rigor to his argument by spelling out measures and indicators for social mobilization and undertaking a fictional data experiment to illustrate model predictions. This portion of the argument really is insignificant but if you are curious I encourage you to examine it in your free time. Ultimately, the most important thing to note about Deutsch’s argument (and possibly Inkeles’) is that it contributes to the literature by defining and specifying social mobilization as the key component of modernization and by suggesting concrete ways to measure modernization through social mobilization. In this way, his article encourages scholars to view modernization as a complete, independent process that can exert powerful influence on other topics of interest such as social organization and politics.

Ronald Inglehart and Christian Welzel, Modernization, Cultural Change, and Democracy: The Human Development Sequence (New York: Cambridge University Press, 2005), Chapters 1-2 (pp. 15-76).

Yet again, this is another piece that looks at the effects that economic modernization and industrialization has on society’s value orientations and cognitive development. Basically the author’s note that 1) modernization has exerted powerful influence on society and 2) the way in which it has done so, is not as classical modernization theory would predict. As a result they attempt to explain certain misconceptions and anomalies of modernization with respect to its effects on value orientation and change. To begin the authors higlight three problems with classical modernization theory: 1) societal transformation (in contrast to the predictions of Deutsch?) has not been deterministic, 2) religion has not died out as predicted, 3) cultural modernization does not appear stagnant or linear and can be reversed, and 4) modernization has not led to “westernization” of society. Thus they propose that an accurate theory or modernization (or its consequences) should make sure to account for the persistence of traditional cultures and religion, allow for changes/reversals in the process of modernization, and allow for distinct cultural paths of modernization that are distinct from those seen in the West. Of course the theory that they advance does all of this and is presented below. The main purpose of their theory is to develop a framework for understanding how cultural change follows from economic change (namely industrialization) in a way that is consistent with their suggested modifications to modernization theory. They argue that cultural change happens in two dimensions: first, industrialization triggers a move towards secularization as God is no longer needed to rationalize environmental phenomena (for example the sun rising) and as people gain more control over nature. While this change is important for making society more secular-rational, it does not imply that society will be any less bureaucratic or rigid, as strict hierarchical systems still dominate the industrial world (note the factory system). Change from this rigidity to a more self-expressive value system therefore arises with post-industrialization. In a post-industrial world people can focus less on survival and more on alternative goals because extreme surpluses created by economic development mean that people have enough money such that they can concern themselves with issues beyond food, shelter, etc. Additionally with the rise of public education and mass communication, there is more informational autonomy in society (people can access information and make independent decisions). All of this will eventually lead people to form opinions about self-expressive concerns such as concerns about gender equality, human rights, etc. Additionally the excesses of modernization will lead people to reconsider God as a way of finding meaning and purpose in their lives. Thus you get the movement from survival orientation to self expressive orientation. It is important to note that these changes are cumulative and happen slowly over time. They note that the institutionalization of these changes usually happen via elites, but always reflects an undercurrent of change that already exists in society. Furthermore, change is not always pleasing to everyone so it is possible that those who are alienated by change can halt or even reverse change processes. Beyond explication of the theory, there is a second section where the authors propose several empirical tests of their results. I will not get into the empirics here because they are unimportant but it will suffice to say they find evidence supporting their claims that secular-rational value change happens during industrialization and that self-expressive value changes happen in the post-industrial period. Additonally, they find that cultural zones have a susbstantive impact on value change, thus explaining why certain societies continue to be traditional despite widespread economic modernization.

Immanuel Wallerstein, “The Rise and Future Demise of the World Capitalist System: Concepts for Comparative Analysis. Comparative Studies in Society and History 16, No. 4 (September 1974), pp. 387-415.

Unlike all of the works described above, Immanuel Wallerstein’s work focuses strictly on the economic (and perhaps very limitedly political) consequences of modernization by emphasizing the changes that it produces in the world economy. Most basically, Wallerstein examines how modernization acts to organize states in the world economy and shapes their economic and political interactions with one another. To begin, Wallerstein posits that the world economy is dominated by a single system–capitalism. He distinguishes between two forms of capitalism, namely agricultural and industrial capitalism, but contends that all states in the world, belong to a capitalist world economy (i.e. the dominant more is capitalism). Within this system, a major consequence of economic modernization was the reorganization of states into three distinct categories: core, semiperiphery and periphery. He argues that some of this reorganization is purely historical accident: certain states were better able to industrialize than others (these became the core) and some only partially industrialized (semi-periphery) while still others could only maintain basic agricultural production (periphery). Thus a differential and hierarchical world system was created wherein the core countries exploited the resources (raw materials, cheap labor, etc) of the periphery to extract economic surplus. Note that the semi-periphery is crucial to maintain the system because it acts both as an exploiter (of the periphery) and the exploited (by the core). Because the semi-periphery holds down the periphery in this way it allows the system to maintain its hierarchical dynamic and ensures that the periphery does not revolt against the core. Also note that stability is reinforced by political peculiarities of the system. Interestingly by virtue of being core states, the core have strong military capabilities that allow them to maintain the hierarchy. After explicating these features of the world system, Wallerstein then goes through historical explanation about how we have moved through the stages of the world capitalist system and how these stages reinforce the strength of this system. These stages are: 1) The crisis of feudalism and the geographic expansion of the division of labor (conquest movements in Europe) that created a European world economy; 2) System wide recession that forced retrenchment of certain states and allowed England to emerge as the only core state, 3) Industrial capitalism that led to further expansion of world economy into the rest of the globe (Africa, Asia and Latin America usually through colonial conquest), and creation of semi-periphery as other countries moved to industrialize; 4) Withdrawal of certain semi-periphery states during period of economic decline into social systems which allowed the to strengthen semi-peripheral status and thus bolster the stability and functioning of the capitalist system. To be honest this last portion of the argument seems somewhat irrelevant to the point that modernization has produced major changes in the world economy, but its useful to the extent that it explains how certain anomalies within his capitalist paradigm (socialist states) actually feed into and strengthen the capitalist world economy.

Barrington Moore, Social Origins of Dictatorship and Democracy: Lord and Peasant in the Making of the Modern World (Boston: Beacon Press, 1966), chapters 7-9 (pp. 413-483)

In a change from all of the works we have explored previously Barrington Moore’s attempt to explore the consequences of modernization focuses on the effects that economic modernization has had on producing differential political outcomes. He argues that the three forms of political systems that we witness today (that is, his today…1960-something)–democracy, fascism, and communism–arise as consequences of particular paths of economic development witnessed in a society. Akin to Marx, he relates the development of these political arrangements to the arrangement of certain class structures in society. Specifically the following relationships between classes in society produce democratic, fascist, and communist systems:

  1. Strong middle class (agrarian entrepreneurs and urban bourgeois) + weak aristocracy -> liberal democracy;
  2. Weak middle class + strong aristocracy (industrialists and landed aristocracy) -> fascism;
  3. Weak middle class + weak aristocracy + strong peasantry -> communism.
    Furthermore the arrangement of class structures is shaped features of economic modernization: commercialization of agrarian economy and demands for revolutionary change. The following relationships predominate:
  4. Commercialization + revolution -> democracy;
  5. Commercialization + no revolution -> fascism;
  6. No commercialization + revolution -> communism;
  7. No commercialization + no revolution -> political stagnation and decay.
    To state his theory succinctly: economic modernization changes the structure of the national economy can shift class structure/alliances by creating shared interests and opportunities for alliance or by strengthening certain classes vis-a-vis others. These class alliances have serious consequences for political development. Specifically the course of political development in a country is determined by 1) the strength of the middle class relative to the aristocracy or peasantry and 2) dynamics between classes (or cross-class alliances) which dominate politics.

Samuel Huntington, Political Order in Changing Societies (New Haven: Yale University Press, 1968), pp. 1-92, 192-263

From 1998 Comparative Politics Summary Compilation
Huntington advances a institutional framework for distinguishing between traditional societies and modernizing ones. He makes a distinction between what is “modern” and what is “modernizing,” calling attention to the destabilizing effects of modernization. Within this, he presents a theory which links success in modernization with political insitutionalization which structures political participation. As modernization engenders social mobilization, a society must adapt its political institutions and increase the level of institutionalization to cope with this increase in participation, aspirations, and demands. Thus, Huntington stresses the degree of government rather than the form (although a more institutionalized government is better than a less institutionalized one). [degree of government speaks to form in ancient formulations and in particular to the recognition of mixed polities providing stability.] Huntington’s first chapter focuses on these issues of modernization, institutionalization, and participation through the lens of a government who governs–that is, provides structure for the deployment of politics. The idea of a developed political community is central. And in order to have this, a society must have adequate and adaptive political institutions. Huntington calls attention to the false notion prevalent post WWII that posited a strong economy as the link to a stable polity (see p. 6). He argues, rather, that a strong, well-developed polity is a separate endeavor on which the economic changes which bring prosperity often depend. His argument is that the political gap between developed and underdeveloped polities is as much a problem as the economic gap (see p. 2). The U.S., through its historical inheritance of already developed British institutions and “equality,” tends to forget this. In the United States, government is about limits rather than consolidation which is the key to effective and stable governments in the process of modernization and development (see p. 6). Huntington adheres to the view that a robust civil society with partial associations and, in his language, political institutions capable of adapting themselves to change, maintain stable political communities. He calls attention to the violence and instability that ocurred in various countries between the years 1955 and 1962, whereas previously there had been none (p. 4). His thesis is that this violence and instability were the the result of “rapid social change and rapid mobilization of new groups into politics coupled with the slow development of political institutions” (p. 4). As Tocqueville asserted, the “art of associating together” has to grow proportionately to equality of participation. Karl Deutsch introduced the idea of social mobilization as the overall change which occurs in societies moving from traditional to modern ways of life (“Social Mobilization and Political Paricipation” in Finkle and Gable, Political Development and Social Change). Social mobilization is “the process in which major clusters of old social, economic, and psychological commitments are eroded or broken and people become available for new patterns of socialization and behavior” (p. 385-386). Social mobilization increases needs, usually for government services (p 391). It is when the government fails to develop an institutionalized apparatus, which extends in some form into the civil society, responsive to these needs that Huntington’s argument comes into play. Both Deutsch and Huntington recognize to potential alternatives of consolidation and decay which social mobilization produces (see Deutsch p. 395). Huntington takes this insight and develops his theory of correlations between institutionalization and political participation to strengthening or decay of political communities. Political instability and disorder results from the lack of an adequate level of institutionalization for the level of political participation generated by social mobilization and the new consequent needs. Institutions are the behavioral manifestation of the moral consensus and mutual interest (p. 10). In more complex societies, the level of institutionalization and diversification is also higher to sustain a developed political community. There is also the matter of scope (p. 12) of incorporation into the political community of different groups and sectors. The adaptability of institutions has to do with 1) chronology: how long they’ve been around and 2) generation: change in leaders and 3) functional terms: functional adaptability. Huntington advances a theory which differentiates between praetorian and civic polities which corresponds roughly to the ancients idea of polities which look to the common good (public interest) and those that look toward the private interest. Praetorian politics is generally characterized by systems which exhibit low levels of political institutionalization and high mobilization. This creates a dynamic of group against group rather than a structured perceived public interest. Groups are thus mobilized without becoming socialized into politics (p. 83). In the civic polity, the price for authority is limitations on what resources may be used for. The distinguishing characteristic of modern societies is the structured mobilization of the masses. Herein lies the importance of the party. In societies where institutions are weak the society reflects the party instead of the party reflecting society.

Political Economy of Development

This literature focuses on economic development, and in particular the questions: how does economic development happen? What forces stimulate or retard economic development? And whether and how economic development differs by country or by timing of development?

W.W. Rostow, The Stages of Economic Growth: A Non-Communist Manifesto (Cambridge: Cambridge University Press, [1960] 1990), pp. 1-12

Rostow presents a generalizable history of the progression of economic development. Rostow describes five stages of economic development: 1) traditional society, 2) preconditions for take-off, 3) take-off, 4) drive to maturity, 5) high mass-consumption. Each stage is characterized by its overall level and growth of productivity as well as its sectoral composition. Rostow’s theory implies that there is one set path of economic development, which all counties will at some point progress along. The transition from stage to stage, which entails changes in the composition of leading sectors, results from changing supply (e.g., technology, entrepreneurship) and demand (individual elasticity to price and income), which in turn stem from individual agency, changing private tastes, changing social norms, and political decisions. These socio-political factors are influenced by “history” broadly inclusive of social structure, values, political institutions, geography and resources, and international economic linkages (Rostow does not focus on identifying specific causal factors). Britain is the first to progress along this development path and does so “endengously” because of favorable conditions. Rostow suggests that counties moving along the stages of growth after Britain may be catalyzed by external political pressure and may move from stage to stage in a more directed (government-led) manner.

Gerschenkron, Alexander, Economic Backwardness in Historical Perspective (Cambridge: Harvard University Press, 1962), pp.5-30

Gerschenkron examines the development trajectory of late European industrializers (Germany, France, Russia) in effort to provide lessons for non-European late industrializers. Premised on the view that the “early” English model of economic development can no longer be replicated. For late industrializers, industrializes depended on availability of opportunity (e.g., agriculture not too profitable) and removal of institutional obstacles (serfdom). Success of late industrializers depended on fast-paced, large scale, multi-sector industrialization using existing technology and industrial labor. In order to facilitate this type of late industrialization, modern banking, the state, and ideology played crucial roles. German and French banks provided long term credit and management intervention that drove industrialization in these countries. The state in Russia, because of its history of military conflict, led economic development. State-led development went in fits and starts, entailed large upsurges that required government oppression, and resulted in stagnation following periods of fast growth. In all counties, some ideology was required to instill faith that “progress” was desirable, and capitalism was too uninspiring to provide this ideology, so socialism and Marxism substituted.

Robert H. Bates, Markets and States in Tropical Africa: The Political Basis of Agricultural Policies (Berkeley: University of California Press, 1981), pp. 1-105

According to Bates, state sponsored capitalism among African countries generally failed to create economic development, instead enriched elites and created incentives for corruption and clientelism. State sponsored capitalism diverted resources from agriculture to industrial and manufacturing. This entailed the taxation of rural farmers and the transfer of revenue to industry in towns. This also included protecting urban interests at the expense of rural farmers by increasing the prices farmers had to pay for urban goods and decreasing the prices farmers received for their products (achieved by overvaluing currency, creating monopsonies, government entry into commodities market, subsidized inputs. These government actions created incentives for corruption, e.g., farmers bribe local bureaucrats for economic protection. As well, state-sponsored capitalism allowed for political marginalism by using markets to fragment rural opposition and instituting pork barrel politics for supporters. Not all states were able to exploit the rural class. Countries with fewer products and larger farms were able to organize and lobby for more favorable policies.

Johnson, Chalmers, MITI and the Japanese Miracle: The Growth of Industrial Policy, 1925-1975 (Stanford: Stanford University Press, 1982), pp. 3-34

Johnson attributes the “miracle” of Japan’s economic growth to the successes of its developmental state. The developmental state or a “plan rational” system, which is a characteristic of late industrializes, sets economic goals and actively works to enhance competitiveness. Against a agreed upon set of overarching goals (e.g., fast-paced growth), an elite bureaucracy intervenes in the design of the overall industrial structure and the operations of specific firms. The Japanese development state and plan rationality were born during the Meiji restoration (1868). Plan rationality initially entailed direct state ownership, but when this led to corruption, bureaucracy, and ineffective monopolies, plan rationality shifted to government collaboration with private firms (zaibatsu). Post-war / during occupation, firms increased and labor force grew. Johnson argues that Japan’s plan rationality (along with flexible labor supply, access to international markets and technology) allowed Japan to shift its economic structure from fibers / textiles to machinery / metal products, and that this shift underlay Japan’s economic miracle. The Japanese development state was constantly learning and adapting (as opposed to corrupting) because the ultimate purpose of the development state was political and rooted in nationalism–the desire of late a late industrializer to become strong.

North, Douglas C., Institutions, Institutional Change, and Economic Performance (Cambridge: Cambridge University Press, 1990), pp. 83-104, 107-117

Institutions as defined by North are rules of the game, humanly devised constraints that structure incentives in human exchange. The include formal rules, and informal extensions and qualifications such as customs, tradition, routines. The key question that North seeks to answer is why have societies, polities, and economies diverged? And in particular, why do poor performing economies persist? An ideal world that starts with different endowments, capabilities, climates, problems should converge as the cost of information falls. Economic development has not converged because institutions, which are the underlying determinant of long-run economic performance, have not converged. Institutions do not converge because history matters and constrains future. History matters because 1) institutions have increasing returns (e.g., once tech industry adopted QWERTY, it never changed even though it was not the most efficient) and 2) markets are incomplete, information feedback is fragmented, and transaction costs are high (this lead to different subjective models). Institutions change can come from change in relative prices (exogenous shock or endogenous entrepreneur) or from change in preference, ideas, tastes.

Krueger, Anne. 1990 “Government Failures in Development,” Journal of Economic Perspectives.

Krueger argues against the view that government should facilitate economic development and compensate for market failure because governments intervention can lead to economic failure. There are failures of commission where government create inefficient and wasteful public enterprises (e.g., marketing boards, banks), There are failures of omission in face of increased costs, overvalue exchange rates, domestic inflation. These failures often lead to corruption. The government should focus its interventions on areas where it has a comparative advantage: infrastructure, rule of law, public services. Government intervention is never costless because they drain scare administrative resources and can take on a life of their own. Especially problematic are policies that create something to be allocated at less its actual value and policies that have identifiable beneficiaries or victims. Governments should aim at transparency and choose policies that force tradeoff in administration and execution in order to minimize rent-seeking.

Przeworski, Adam, and Michael Alvarez, Jose Antonio Cheibub, and Fernando Limongi, Democracy and Development: Political Institutions and Well-Being in the World, 1950-1990 (Cambridge: Cambridge University Press, 2000), pp. 142-179.

Engages in the debate of whether democracy is good or bad for development. Przeworski et al. find that political regime has no impact on the growth rate of total income. For example, although average share of investment in GDP much higher in democracies, after controlling for income, regime has no effect on investment. As well, the average rate of growth in countries that did and did not experience regime transitions are similar. Przeworski et al. argue that poverty is persistent and trumps regime. Poor countries stay poor, but among wealthy countries, the driver of growth differs between democracies and dictatorships. Dictatorship obtain growth through labor-intensive and labor-exploitative means, while democracies achieve growth through return to technology. As a result of this, Przeworski et al. argue that growth in dictatorships may be more transitory.

Alice Amsden. 2001. The Rise of “The Rest”: Challenges to the West from Late-Industrializing Economies. (Oxford: Oxford University Press, 2001), pp. 1-17, 125-160.

Amsden defines economic development as the “process of moving from a set of assets based on primary products, exploited by unskilled labor, to a set of assets based on knowledge, exploited by skilled labor” (2). Amsden’s main argument is late (non European/N. American) industrializers adopted a different strategy to achieve economic development, which depended on government led industrial development. North Atlantic counties that achieved economic development did so because they industrialized in tandem with technological change, which automatically led to “three-pronged” investment. For counties to achieve economic development after technologies were already in place, counties had to overcome “knowledge-driven entry barriers.” The “rest” overcame the knowledge deficit through “reciprocal control mechanisms” that pushed manufacturing. Control mechanisms are “institutions to discipline economic behavior,” i.e., developmental state that distributed subsidies, tax/tariff/duty benefits, credit. Control mechanisms are reciprocal because incentives/benefits were tied performance monitor and policy constraints, e.g., products must be exported, parts locally sourced . Amsden mentions divergence within the rest as some counties invested more in proprietary national skills due to their type of early manufacturing experience.

Jeffrey Sachs, “Tropical Underdevelopment,” NBER Working Paper (2001), pp. 1-40.

Sachs makes his well-known argument about the dominant role of ecology in economic development. He argues (with limited evidence) that geographies with temperate climate and access to sea trading are advantaged in economic development, whereas land-locked geographies with tropical climate are disadvantaged. For Sachs, the driving force of economic development is continued technological development (31). Ecology is crucial to technological development because temperate zones had higher technological productivity compared with tropical zones pre-modernization (due to soil/air/temperature, burden of disease, energy resources). This early advantage was amplified during modernization because technologies did not diffuse from temperate to tropical zones, because technologies had increasing returns to scale, because tropical zones has slower urban and demographic transitions, and because temperate colonized tropical zones. Sachs argues against the impact of colonization alone by stating that tropical Africa had low standards of living before colonization and that post-colonial tropical Latin America has not done well. Sachs also argues that linkages to Europe and the legacy of capitalism do not explain divergent development since tropical European colonies have not done well while temperate East Asia has done well.

Acemoglu, Darren, S Johnson and J Robinson, “Reversal of Fortune: Geography and Institutions in the Making of the Modern World Income Distribution,” Quarterly Journal of Econ 118 (2002): 1231-94

Acemoglu et al. find that there is a negative relationship between economic prosperity (measured by urbanization and population density) in 1500 and per capita income today. This is what they term “reversal of fortune.” Since theories of economic development based on geography and ecology cannot explain reversals in development, the authors present an institutional theory to explain causes of contemporary differences in economic development. Countries that were relatively prosperous in 1500 (and which by definition had larger populations) were subjected to elite-centered extractive institutions by European colonizers because it was more profitable for the colonizers to extract (e.g., force native population to work in mines, milk existing taxation system). Countries that were relatively poorer in 1500 (and which were sparsely populated) were endowed with institutions of private property because Europeans settled in these regions. These two types of colonial institutions led to divergent paths of development once industrialization in the late 1800s/early 1900s occurred because extractive, elite-centered, institutions hindered technologically-based growth.

Dani Rodrik, Arvind Subramanian, and Francesco Trebbi, “Institutions Rule: The Primacy of Institutions Over Geography and Integration in Economic Development” Journal of Economic Growth 9 (2004), pp. 131-158

Rodrik, Subramanian, and Trebbi test three competing explanation for divergence in economic development: 1) geography (measured by distance to the equator), 2) international trade integration (measured by volume of trade flows), and 3) institutions (measured by perception of property rights protection and rule of law). The authors use instrumental variables to determine causality and find that institutions have the most influence on current incomes, and that institutions also have a positive effect on trade integration. Since Rodrik, Subramanian, and Trebbi’s measure of institutions is based on perception, they conclude that credibly signaling property rights protection appears more important than enacting formal, legal property protection.

Glaeser, Edward, Rafael LaPorta, Florencio Lopez de Silanes, and Andrei Shleifer, “Do Institutions Cause Growth?” Journal of Economic Growth 9 (2004): 271-298

Glaeser et al. argue that human capital is a more basic driver of growth. Human capital leads to good policies, good leaders, and effective institutions, which enable countries to escape poverty. Glaeser et al. critique institutional accounts of economic development by critiquing how institutions are measured and the instrumental variables used to identify the causal effect of institutions.

The State and State-Building

Concept Map

The early literature on the State and State-Building can be viewed as a critique to Modernization theory, namely that modernization cannot occur without a strong state but that a strong state only develops under specific conditions. The summaries below address the concept of the State (Krasner, North, Geddes), the fiscal-military model of state building (Tilly, Bates, Ertman, Herbst), amendments to the fiscal-military model (Centeno), and alternative modes of state building (Slater).

Charles Tilly, “War Making and State Making as Organized Crime” in Peter Evans, Dietrich Rueschemeyer, and Theda Skocpol, Bringing the State Back In (Cambridge University Press, 1985), pp. 169-191

The argument in brief: Power-holders seek to expand and defend their territory, so they made war on other power-holders; to be successful at war, they needed an efficient state (for taxation to fund war) and capital accumulation (to create a tax base); thus they invested in institutions to achieve these goals. Therefore, in this way, war is the catalyst for the development of a competent bureaucracy and a growing economy. By the time the drive for war had died out, state-makers had developed a durable interest in promoting accumulation of capital. Additional notes: Chief activities of the government are war, repression, protection, and adjudication. Explanation of differences in bureaucratic structure of successful state: the more costly the activity, the greater the organizational revenue: so, the smaller the pool of resources and the less commercialized the economy, the more difficult is the work of extracting resources to sustain war and government activities (e.g., in France, there was thus a more extensive fiscal apparatus). Regarding the applicability of the theory of development countries today: today’s developing countries didn’t go through the internal process of forging a viable military (thereby forging a viable state); outside military and aid supports make third-world leaders powerful and unconstrained in their actions, since they do not have to form alliances with domestic groups to stay in power.

Robert Bates, Prosperity and Violence (2nd edition) (WW Norton, 2009), Chapters 3-4 (pp. 34-66).

The argument: Similar to Tilly’s argument, in that the war is the key driver of state development in Europe. Bates is unique in emphasizing the role of cities in war and state creation–he argues that as cities began to accumulate wealth, they sought protection; war-makers who allied with the cities could offer that protection, and in turn were given access to a growing tax base, enabling further conquest. Because it is difficult to force the city’s revenue-earners to contribute to the ruler’s treasury (in the countryside, one can sack a castle and steal to gold; in the city, merchants will simply flee or bank abroad) bargains must be stuck : here we get the rise of parliaments. Rulers exchange policy concessions and limits to their power in exchange for stable state revenues (which come from taxes). Bates sums up the general idea: “Governments pursued policies that promoted the growth of the economy and the rise of parliamentary institutions not because they wanted to but because they had to, the better to secure the resources with which to fight” (56). Regarding the applicability of the theory of development countries today: Bates’s analysis of their poor development is similar to Tilly’s. According to Bates: “In the postwar period, governments in the developing world derived a major portion of their revenues from international transfers; they had little reason, therefore, to bargain with their citizens….The availability of resources from abroad weakened the incentives for executives to elicit support from their citizens–or for citizens to challenge the policies of their governments” (65).

Jeffrey Herbst, States and Power in Africa: Comparative Lessons in Authority and Control (Princeton: Princeton University Press, 2000), pp. 12-31

Herbst presents a “political geography” argument about state formation: Regarding Europe, Herbst notes that the reason that these states competed with each other for territory was that territory was scarce (or, the ratio of people to land was relatively high, and states fight for the commodity in short supply). In Africa, the situation was reversed. Population densities were quite low, and so (pre-colonial) African states did not embark on the European path to development. Colonial cities in Africa were developed with “colonial interests” (Herbst does not specify what these are) in mind. They are located on the coast, rather in than in the center of the country, which implies that they did not seek to build the kind of state-integrating infrastructure (and institutions) that had been built in Europe. Building infrastructure is expensive, and particularly so in Africa (where territory is vast, and populations widely dispersed); therefore, post-colonial leaders did not take on the task either, and post-colonial politics remained “urban affairs.” In the 1970s and 1980s, a severe economic downturn caused the decay of whatever infrastructure/institution building that African leaders had managed to build, thus making weak states even weaker. Furthermore, the respect given to colonial borders guarantees state sovereignty in the international system: “African states have never had the security imperative to physically control the hinterlands in the face of competition from hostile neighbors” (21).

Miguel Centeno “Blood and Debt: War and Taxation in Nineteenth Century Latin America,” American Journal of Sociology 102 (6) 1997, pp. 1565-1605.

Centeno presents a critique of Tilly’s “war makes the state” argument. “Appreciation of historical specificity and structural conditions is vital for the production of truly generalizable models of state development. Only some wars built states, only some states were built by wars” (1568). Plenty of states were destroyed by war. Three critical prerequisites for institutional development aided by war: 1) states must be forced to turn inward in order to meet financial needs, 2) adequate administrative mechanisms must be in place to manage the explosion in both revenues and expenditures, 3) the central state must have already established sovereignty over its territory and must be supported by enough local actors so as to make domestic extraction profitable. In other words, war may provide a stimulus for growth; and war may strengthen states that are already fairly well organized (politically and economically); but in weak states, war leads chaos and defeat. In Latin America, some results of war were the same as those in Europe: destruction of the loser; increase in size of government for the winner; economic stimulus for the economy. Regarding taxation, however, states continued to rely on taxes on trade, rather than develop a strong, internal fiscal apparatus. The point is that war created neither a new type of economy in Latin America, nor a new type of taxation infrastructure. This is because (relating to list above): 1) the Latin American economy was oriented outwards (towards trade with Europe), 2) the wars in Latin America occurred too soon after independence, when institutions were still quite weak, 3) “no class was powerful enough to impose its domination, and no state was strong enough to enforce its control,” (1598). Conflicting class interests reinforced the state’s incentive to rely on external trade as a source of revenue, as antagonizing domestic groups would only stir up trouble. (Presumably domestic repression was troublesome in Europe as well, but trade with an outside set of super-powers wasn’t an option then.)

Thomas Ertman, Birth of the Leviathan: Building States and Regimes in Medieval and Early Modern Europe (Cambridge University Press, 1997), pp. 1-34.

Ertman does not dispute Tilly’s theory of state-building through war-making, but he is interested in what Tilly finds largely unproblematic–that is, the qualitative differences between the governments that developed in the European states. Breaking with the “conventional” view of dividing the European states into two types–absolutist regimes with large state bureaucracies and constitutional governments with patrimonial administrations–Ertman separates the type of regime from the type of administration. Taking a hard look at the full range of European cases, he finds that constitutional governments can operate through an extensive bureaucracy (as in England); and that some absolutist governments rely on patrimonial administrative structures (such as France). Reviewing the distribution of the European states among the four categories (absolutist-bureaucratic; absolutist-patrimonial; constitutional-bureaucratic; constitutionalpatrimonial), Ertman finds that geography cannot explain their positions. Among the landed are Hungary (constitutional-patrimonial), France (absolutist-patrimonial) and Germany (absolutist-bureaucratic). Facing the same problems of collecting land taxes and defending the landed estates, they should all have developed the same type of government and administrative apparatus. They did not, and so Ertman offers an historical explanation. It is a complicated argument, but its broad contours are shaped by:

  1. Legacy of the Holy Roman Empire on the structure of local government (strong local notables and a fragmented political landscape in former HRE; more rational administrative units and freer population in areas only lightly touched by HRE).
  2. Administrative, technological and financial tools available to the state at the time of state-building (the more advanced the tools, the fewer concessions the central government needed to make to the notables and the broader population).

Barbara Geddes, Politicians Dilemma (University of California Press, 1994), pp. 1-19

Geddes supports the idea of state autonomy (arguing against the idea that the state is just a tool of the most powerful social group), but notes that policy cannot be explained by hard-to-observe state “preferences.” Furthermore, Geddes critiques the idea of “state” as unitary actor, arguing that states are comprised of many actors, often with competing interests and agendas. She argues that governments consist of rational individuals who seek to maximize career success (ie, stay in office). It is the institutions in which they work and seek advancement that shape their incentives, and so incentivized, the officials shape policy. The “politican’s dilemma” explains why the development of state capacity is so difficult: institutional incentives for politicians often militate against the policies they should enact; short-term political needs are at odds with long-term economic/developmental needs. Geddes gives the example of Chile under Allende, where the copper mines where nationalized and subsequently mismanaged, wrecking the national economy. It was not that there was a lack of ability to manage the mines well, but rather that political constraints forced Allende to make political appointments to management posts instead of hiring those with the appropriate skills. Long-run economic well-being was sacrificed for immediate political stability.

Dan Slater, “Can Leviathan be a Democrat? Competitive Elections, Robust Mass Politics, and State Infrastructural Power.” Studies in Comparative International Development 43:4 (December 2008), pp. 252-272

Slater argues that state-building can occur by democratic means–specifically, through the mobilization of mass political parties. The idea is that:

  1. State infrastructure is built, state presence expands, and local strong-men are undermined when the government initiates a massive voter-registration drive and administers elections.
  2. Political mobilization of the masses (along with competitive elections) will create strong demand for better government services, and governments, needing the votes, will be obliged to provide them - so we get land reform, public housing, high quality education, transportation infrastructure, etc.
    Note that a democratic construction of the Leviathan requires not only elections, but competitive mass elections. Slater is arguing against the emphasis placed on parliaments by students of European state-building: “For most East Asianists, the elite societal figures who gain access to the state via parliamentary elections have been seen more often, not as a necessary check against state predation, but as predators themselves,” (255). Thus the need for mass politics in Slater’s analysis.

Anna Grzymala Busse, “The Discreet Charm of Formal Institutions: Postcommunist Party Competition and State Oversight,” Comparative Political Studies (December 2006), pp. 1-30.

Using post-Communist politics in Eastern Europe (where some countries emerged as respectable democracies while others did not) to motivate the puzzle, GB aks why governments would choose to constrain themselves, self-imposing measures of democratic transparency and checks to power. The answer lies in the presence or absence of “robust competition” and “robust opposition” which, when present, cause the ruling party to limit its own power because:

  1. Robust competition ensures turnover in government; the ruling party wishes not to constrain itself, but its successor and opposition.
  2. Robust opposition ensures that the opposition party will highlight the ruling party’s abuses of power, thus arousing an electoral backlash against the party in power. the ruling party does not want to upset the voters and will therefore behave as virtuously as possible while in office.
    When electoral turnover is unlikely, or when the opposition is too weak or peripheral to arouse popular anger against the government, the ruling party is likely to entrench itself in office and abuse its power.

Robert H. Bates, When Things Fall Apart: State Failure in Late-Century Africa (Cambridge University Press, 2008), pp. 3-6, 15-29, 97-139.

Bates argues the opposite–that the advent of multipartism led to increasingly abusive governments. Using post-colonial African countries (the ones that failed, which, in fairness, is not a small number) to motivate the puzzle, he asks about the conditions under which states establish order, or prey upon their people. “The possibility of political order rests on the value of three variables: the level of public revenues, the rewards from predation, and the specialist’s rate of discount” (20). These correspond to:

  1. As state revenues decline, so do the rewards of public service. “those who control the means of violence”–ie, the military–would “therefore be more likely to turn to predation,” (25).
  2. Presence of natural resources (eg, diamonds, oil, etc) increase the rewards to predation relative to the slow-growing rewards of investment in domestic economic growth.
  3. The advent of multi-party elections make the incumbent less secure. “They would therefore find the modest rewards that accrue to political guardians less attractive, and the fear of future punishment less daunting, increasing the temptation to engage in predation” (27). (This is the part that directly contradicts Grzymala-Busse.)
    In sum, the loss of revenues in the 1970s and 1980s; the loss of political monopolies in the 1990s; and the availability of natural resources all encouraged state predation because state-building (and the maintenance of law and order) became less profitable than looting. African states failed when predatory behavior on the part of the state provoked rebellion.

Institutions and Institutional Analysis

According to Douglas North (1990), institutions are “rules of the game in a society…humanly devised constraints that shape human interactions. In consequence they structure incentives in human exchange” (3). The concept of institutions is one of the most important in modern political science—institutions are both an object of study in political science as well as an approach to the study of political science. The summaries below cover works on the institutional approach to the studies of politics, the effects of institutions (e.g., the effect of majoritarian vs. proportional representation electoral systems), explanations for institutional design (e.g., a solution to a collective action problem, the result of critical junctures and path dependence), and explanations of institutional change.

[Approaches] Peter A. Hall and Rosemary Taylor, “Political Science and the Three New Institutionalisms,” Political Studies, 44 (December 1996)

Under the heading of new institutionalism, there are three modes of theorizing: (1) historical institutionalism, (2) rational choice institutionalism, and (3) sociological institutionalism. Historical institutionalists define institutions as “formal or informal procedures, routines, norms and conventions embedded in the organizational structure of the polity or political economy” (p.938) and in reaction against structural-functionalist explanations focus on how “institutional organization of the polity and economy structures conflict so as to privilege some interests while demobilizing others” (p.937). Under the historical approach, one can further distinguish calculus approach (behavior is affected through changing information structures, enforcement mechanisms etc.) and a cultural approach (bounded rationality, worldview and habits determine behavior). Emphasis on path dependency and critical junctures. Rational choice institutionalism began with the possibility (social choice theory) of cycling problems (unstable majorities), to which institutions appear as a solution. (There was a clear “foundational puzzle”.) Generally, institutions appear as results of human intentionality and strategic answer to collection action problems, an instrument to realize gain from cooperation. Sociological institutionalism sees institutions as culturally-specific practices that are transmitted not for any particular superior rationality. Indeed, the distinction between culture and institution itself blurs. Related to Hall/Taylor are the chapters by (1) Weingast, describing in detail rational choice institutionalism and (2) Pierson/Skocpol, explaining the idea of historical institutionalism.

[Approaches] Paul Pierson and Theda Skocpol, “Historical Institutionalism in Contemporary Political Science,” in Ira Katznelson and Helen V. Milner, eds. Political Science: State of the Discipline (New York: W.W. Norton, 2002), pp. 693-721.

What makes historical institutionalism a coherent genre, a somehow unified research approach, what is its core research strategy? Historical institutionalism is one out of three approaches in empirical political science (besides survey-based behavioralism and rational-choice modeling), particularly apt at explaining long-term processes and identifying critical historical junctures. Institutionalists (1) address big questions with real-world relevance (2) “take time seriously, specifying sequences and tracing transformations and processes” (p.695) and (3) analyze “macro contexts and hypothesize about the combined effects of institutions and processes” (p.696). The range of topics addressed by institutionalism is huge: “social movements, the development of the modern state, ‘, regime transformations” (p.697) and often includes a normative concern. “Taking time seriously” seems to be a most characteristic property of historical institutionalism: Historical process tracing (which reveals causal relationships in a small number of case studies), attention to path dependence (initially small perturbation, eventually huge variation in results; self-reinforcing dynamics; critical junctures), sequences (example p.700: Ertman’s analysis of European states) and conjuctures (=”interaction effects between distinct causal sequences that become joined at particular points in time”, p.702; example Skocpol on revolution) and slow-moving processes. Historical institutionalists are fundamentally skeptical about functionalist explanations. To distinguish historical institutionalism from rational choice institutionalism, the authors point to an inclination in rational choice to model institutions as a solution to collective action problem, whereas historical institutionalists “see institutions as the developing products of struggle among unequal actors.” The authors also agrees with Weingast that there is a “micro” quality to rational choice models and a “macro” quality to institutional models. There is mentioning of the problem of generalizing from small-n studies.

[Approaches] Barry Weingast, “Rational-Choice Institutionalism.” in Ira Katznelson and Helen V. Milner, eds. Political Science: State of the Discipline (New York: W.W. Norton, 2002), pp. 660-692.

Weingast distinguishes between two levels of rational choice institutionalism. The first level, which is the bulk of the literature in this field, investigates how (normally formal) institutional rules affect political outcomes. The second level of analysis, building on the recognitions of first -level analysis, asks why certain institutions are chosen, given that actors know (at least hold beliefs) over the effects of institutional choices. In this second-level analysis, institutions are endogenously explained; this kind of rational choice institutionalism then is closer to historical institutionalism. Rational choice institutionalism at this point is mostly a micro-theory, and the challenge is to turn it into a macro-theory to explain big outcomes like revolutions. As Weingast’s several examples illustrate, rational choice institutionalism is closely associated with formal modeling of the effects of institutions. (Although in the piece North/Weingast, the argument is not formalized, but rather is based on game theoretic ideas without actually doing game theory.)

[Institutional Effects] Arend Lijphart, “Constitutional Choices for New Democracies,” Journal of Democracy (Winter 1991).

How should new democracies in Europe and Latin America design their institutions? Assuming that constitutional choices matter greatly and that we can predict institutional outcomes well, Lijphart very confidently engages in constitutional engineering. To put this piece in context, remember that Lijphart is famous for advocating the “consensus model of democracy” (aka consociationalism, exemplified by the political system of his home country, the Netherlands), as a way to accommodate minorities in democratic systems. In this piece, he makes a similar case for “a combination of parliamentarism with proportional representation” (p.72). He begins by stating three laws that seem to hold true universally:

  1. Majoritarian/plurality systems (“winner takes all”) will produce: (1) a two party system, (2) one-party (as opposed to coalition) governments (3) strong executives.
  2. Proportional representation systems will necessarily lead to: (1) a multiparty system (2) coalition governments (3) power balance between executive and legislative power.
  3. Presidentialism leads to: (1) two party system (2) one-party (as opposed to coalition) governments, BUT (3) executive/legislative power balance.
    There are two big constitutional choices, leading to the 2 by 2 matrix on p. 74: (1) Electoral systems (majoritarian or PR) (2) presidentialism or parliamentarism. In Lijphart’s view, the combination of PR plus parliamentarism is optimal for heterogeneous societies (which is almost all societies), because the combination gives even minorities a voice in parliament, thus including them politics. The way Lijphart evaluates democratic performance (using indicators like economic growth, inflation, unemployment rates, p.80) seems highly contestable. Maybe the most ingenious part of this piece is the way Lijphart not only explains institutional outcomes, but also takes a step back, arguing that in the past, institutional choices by the “old democracies” were made precisely to achieve these institutional outcomes. While admitting that overall constitutional patterns have been “to a large extent determined by geographic, cultural, and colonial factors” (p.74), he remarks that PR systems emerged first “in the ethnically most heterogenous countries” (a point taken from Rokkan), precisely because PR systems are good at accommodating (ethnic) minorities. Secondly, the old elite, knowing they would soon become a minority in the face of a rising working class, found PR a practical system to remain in the political game even after democratic reform. Under plurality system, the old elite would have completely lost its voice. Nice quote from Satori, illustrating Lijphart’s believe that formal institutions matter greatly: Electoral systems are “the most specific manipulative instrument in politics” (p.73). This piece speaks to Dominguez piece in that both authors are concerned about making democracy more consensual (beyond just having the majority rule), but Lijphart thinks formal institutions can achieve that, while Dominguez takes a broader view, with ideas, political habits etc also having a great impact.

[Institutional Effects] Matthew S. Shugart and John M. Carey, Presidents and Assemblies: Constitutional Design and Electoral Dynamics (Cambridge University Press, 1992), Chapters 1-3 (pp. 1-54) and Chapter 13 (pp. 273-287).

To get a sense for the entirety of the book, Warwick’s 1994 one-and-a-half page book review in Comparative Political Studies is useful. Also, in the big picture, Shugart/Carey argue a minority view (that presidentialism is superior to parliamentarism), directed at the majority view (as the one held by Lijphart) that parliamentarism is the way to go. One “trick” that Shugart and Carey are using is to define presidentialism unusually broadly. Western European cases that are used by other authors to show the advantage of parliamentarism are interpreted by Shugart and Carrey as presidential systems. The fundemantal trade-off in constitutional design, according to Shugart and Carey is between efficiency and representativeness. Presidents are efficient in implementing politics and moving beyond the status quo, while assemblies better at representing minority opinions. Presidential systems strike a better balance between the two, because there are myriads of ways of introducing some elements of representative democracy (even in presidential systems there will usually be an assembly as well.) Among the readings, Shugart and Carey strike me as being able to spell out many, far-reaching political consequences of relatively minor variations in institutional design. They do not spend much time explaining where institutions come from (pointing to historical and path-dependent contingencies), instead concentrating on their effects (level 1 analysis in Weingast).

[Institutional Effects] George Tsebelis, “Veto Players and Institutional Analysis,” Governance 13: 4 (October 2000): 441-474.

How hard is it in a given political system to change the status quo? How responsive is a system it to a changing environment? Tsebelis’ parsimonious veto player framework is remarkable for trying to analyze political systems independent of regime type and policy change without regard for policy content. Every time a new political decision (defined as a deviation from the status quo) is taken, there are coalitions, which could block the decision. Definition of the veto player (p.446): “individual or collective actors whose agreement is necessary for a change of the status quo” and the law he finds: “as the number of veto players in a political system increases, policy stability increases.” The trouble begins when you actually try to count how many veto players there are in a given system.

[Institutional Effects] Dominguez, Jorge I., “Free Politics and Free Markets in Latin America,” Journal of Democracy 9:4 (October 1998): 70-84.

Why has democracy in Latin America suddenly become compatible (or even conducive) to free markets, when in the preceding decades popular majorities were seen as hostile to free markets? Dominguez does not seek the answer in formal institutions, but rather in the way politics are done in Latin America since the early 90s. (I would call it a new “political culture”, but the term is avoided throughout.) Executive leaders do not simply implement majority pressures that are hostile to free markets, and their electorates let them get away with it; maybe recognizing the ultimate utility of free markets. The most likely candidate to have driven this new way of running politics has to do with “theoretical and empirical ideas” privileging free market (p.72), that are dominating international discourse. A critical juncture was the depression of the 80s, which made it clear that Latin America needs to change the way of running its politics and its economics to have a chance to develop. Dominguez goes through the cases of Chile, Argentina, El Salvador and Brazil to show that governments in the 90s have negotiated broad-based compromise and super-majorities, including center-right groups inessential to winning a majority. Dominguez account seems very compatible with Helmke/Levitsky’s later “informal institutions” perspective.

[Institutional Effects] Gretchen Helmke and Steven Levitsky, “Informal Institutions and Comparative Politics: A Research Agenda.” Perspectives on Politics 2, No. 4 (2004): 725-740.

The purpose of this paper is to establish a “framework for incorporating informal rules into mainstream institutional analysis” (p.734). Definition of informal institutions: “socially shared rules, usually unwritten, that are created, communicated, and enforce outside of officially sanctioned channels”. (P.727). Figure 1 gives a typology of informal institutions, distinguishing along the dimension (1) the formal institutions are effective (2) and if informal institutions reinforce or weaken formal institutions, leading to “complementary”, “accommodating”, “substitutive” and “competing” informal institutions.

[Institutional Design] Douglass C. North and Barry R. Weingast, “Constitutions and Commitment: The Evolution of Institutions Governing Public Choice in Seventeenth Century England,” Journal of Economic History 49, No 4 (December 1989): 803-832.

This is a functional argument that illustrates the properties of rational choice institutionalism. It explains institutions as a more or less conscious solution to a collective action problem. The problem is that the Crown had confiscatory powers, which in the absence of property rights could be arbitrarily exercised. The absence of economic rights inhibited the economy and made it impossible for the Crown to borrow money (unless obliging people to lend). By establishing institutions that made it harder for the Crown to renege on property rights, greatly enhancing the predictability of the Crown, leading economic security and prosperity.

[Institutional Design] Terry Moe, “Political Institutions: The Neglected Side of the Story.” Journal of Law, Economics, and Organization 6 (1990): 213-253.

The most important sentence of this article is this: “Institutions are crucial means by which the winners win and the losers lose.” (p.214). Moe criticizes conventional institutional analysis -he is referring to rational choice institutionalists- for a neglecting a whole set of issues related to distributional conflicts. Conventional institutional analysis emphasizes how institutions solve coordination problems (as North/Weingast on the role of property rights), moving society as a whole to a more efficient outcome where everyone is better off. Institutionalists rarely analyze how institutions help winners to establish their political dominance. Acemoglu and Robinson, who are writing 16 years later, seem at least partly to help overcome this problem, since they model precisely conflict and distributional outcomes of politics, situations where there are winers and losers. This summary makes the article seem much more coherent than it really is. The agenda of conventional institutionalists, according to Moe, is driven by their methodology, in the tradition of Arrow and social choice theory. One famous problem driving this agenda is the issue of voting results that are cycling: With majority voting over two or more alternatives, there may never be a stable outcome. Institutions stabilize outcomes. By contrast, Moe brings in a whole new set of ideas how to change established ways of analyzing institutions. There is not yet any coherent theory. A few proposals: Institutions distribute public authority (p. 222); and it is quite different to negotiate over the distribution of resources (conventional), but rather over the power to redistribute resources (new). The focus on congress (conventional) should be shifted to the public bureaucracy (new), where interest groups clash in a different way. Instead of studying US only (conventional), move to other democratic systems (new). The article culminates in the “within-coalition problem:” “They [interest groups] want structures that will effectively provide them with benefits, but they also want structures that will protect them from opposing interests-and protect them from the party itself.”

The International Context

Concept Map

The works below examine the interplay of domestic and international political forces. Rogowski and Frieden examine the way in which the international economy impacts the organization of interests in a particular country. Katzenstein, Garrett, Iversen & Cusack, as well as Kaufman & Segura-Ubiergo offer differing perspectives on what governments do, or fail to do, to cope with the domestic and international dimensions of the state. Goldstein and Simmons et al. examine the way in which international laws, institutions, and norms affect the working of domestic institutions. And finally, Tarrow and Keck & Sikkink analyze the activities of communities that operate across national boundaries. Portions of these notes draw from the Harvard Department of Government Comparative Politics Field Seminar summaries of 1998 and 2000.

Ronald Rogowski, “Political Cleavages and Changing Exposure to Trade,” American Political Science Review 81, No. 4 (December 1987): 1121-1136

Rogowski derives his hypothesis from the Stolper-Samuelson theorem, which shows that 1) protection benefits (and liberalization harms) the owners and consumers of factors which are scarce in the particular society, relative to the rest of the world, and 2) protection harms (and liberalization benefits) the owners and consumers of factors which are abundant in the particular society, relative to the rest of the world. Rogowski’’s hypothesis is that “increases or decreases in the costs and difficulty of international trade should powerfully affect domestic political cleavages and should do so differently, but predictably in countries with different factor endowments” (p. 1121-1122). Surges in the volume of trade can result from transportation innovations, and the emergence of an international hegemon (who supposedly makes trade less risky). The only source of diminishing volumes of trade that Rogowski identifies is the decline of international hegemony. The model is based on the following assumptions: 1) the winners will try to preserve the status quo or expand free trade, while the losers will try to gain protection or at least halt the expansionof free trade. 2) The winners will also gain more political influence. 3) There are only three factors, which characterize society: land, labor, and capital. 4) No society can be rich both in land and labor. Assumptions 3 and 4 suggest that there are four types of economies and changes in the exposure to trade have the following effects on political cleavages in each type:

  1. Capital rich (advanced), land rich, labor poor -> class conflict (increased trade will benefit capitalists and land-owners, and harm rural and urban workers, decreased trade will have the opposite effects). Twentieth century US this time exemplifies the first type of economy, hence decreased trade favored only labor, labor became politically powerful and the result was the New Deal.
  2. Capital rich (advanced), land poor, labor rich -> urban-rural conflict (increased trade will benefit workers and capitalists, and harm farmers and land owners, decreased trade will have the opposite effects). Late nineteenth century Britain exemplified this second type of economy, which produced the Liberal Party–a coalition of capitalists and workers, who favored free trade. Twentieth century Germany and Italy fit the second type of economy, hence decreased trade favored only the rural sector, and hurt and radicalized both labor and capital. The results were Hitler and Mussolini.
  3. Capital poor (backward), land rich, labor poor -> urban-rural conflict (increased trade will benefit farmers and land owners, and harm workers and capitalists, decreased trade will have the opposite effects). Late nineteenth century US fits this type of economy, and thus experienced urban-rural conflict, which produced the Populist movement. Twentieth century Argentina, Brazil and other land-rich states exemplified the third type of economy. Decreased trade benefited their labor and urban middle classes against the losing rural elites. The result were Populist coalitions.
  4. Capital poor (backward), land poor, labor rich -> class conflict (increased trade will benefit workers and harm capitalists and land-owners, decreased trade will have the opposite effects). Late nineteenth century Germany fits this fourth type of economy, and thus saw the intensification of class conflict, which produced German socialism and the anti-free trade coalition of land-owners and capitalists. Twentieth century Eastern European states and some Asian states (Japan, China, Vietnam) exemplified the fourth type of economy. Decreased trade debilitated labor and strengthened landowners and capitalists. The result was again fascism (and communism).
    Rogowski suggests that his theory might help explain puzzles such as: preference for statism among late industrializers (capitalists and landowners need a strong state to subdue radical labor), the absence of socialism in the US (socialism appears only in labor rich backward economies under conditions of expanding trade), and the appearance of the Northern coalition in the US Civil War. He admits that some countries may be rich in both labor and land, while others might be poor in both factors. Even if societies which were either rich or poor in all three factors existed, they would not have any cleavages, because the rich societies would unanimously embrace free trading, while the poor societies would concur on protection. Thus Rogowski goes on to explain the two remaining variants. In an advanced economy poor in both labor and land, expansion of trade will produce a protectionist coalition among labor and agriculture pitted against freetrading capitalists (eg., here it goes again, Sweden in the 1930s). In a backward economy rich in both labor and land, expansion of trade will give rise to a free-trading anti-capitalist coalition (eg. Russia until WWII).

Jeffry Frieden, “Invested Interests: The Politics of National Economic Policies in a World of Global Finance,” International Organization, 45 (Autumn 1991), pp. 425-451

Frieden has two objectives in this paper: (1) to analyze what actors benefited or lost from a rise in international capital mobility and (2) to analyze, given a high level of international capital mobility, what are the political interests of various actors. Frieden begins by establishing that international capital has, in fact, increased in mobility. He finds that it has, citing some anecdotal evidence, but shows that there is not full international capital mobility. Borders still pose something of a barrier to capital flow due to uncertainty, increased risk, and lack of harmonization and integration of equity markets, etc. Nonetheless, international mobility of capital assets has increased greatly from the 1970s to the 1990s. He analyzes how increased capital mobility affects national macro-economic policy. Such mobility makes sector-specific policies less effective. For example, protectionist policies are ineffective when barriers are low because foreign producers can easily relocate in the protected market. The largest effect of increased capital mobility, and the consequent financial integration, is on “the effectiveness and the differential distributional impact of national macro-economic policies.” Increased capital mobility creates a situation in which a “country can have at most two of the following three conditions: a fixed exchange rate, monetary policy autonomy, and capital mobility.” That is, high capital mobility creates a trade-off between a countries control of its exchange rate and its autonomy with respect to monetary policy. Frieden reviews three theories of the effects of increased international capital mobility. He argues that neither a portfolio choice nor Heckscher-Ohlin model can fully capture the dynamics at work. More appropriate, he argues, is a specific-factor model. In this model, “the economy is organized into activities (or sectors) to which factors are specific, along with factors that can move freely from activity to activity.” That is, there are multiple industries (sectors) and there are some inputs to those industries that are specific to the industry and some inputs that can be easily applied to different industries (such as unskilled labor). Using this “specific-factor” model, Frieden analyzes the impact of increased capital mobility on various groups. The general results of this model are that as capital mobility increases, capital should move from areas with low interest rates to places with higher interest rates. This causes an equalization of interest rates (they go up where they were low and down where they were high). This change in interest rates effects not only investor, but also producers, because the interest rate is a cost of production when producers borrow. In general, then, increased capital mobility will cause capital to flow from the developed world to the undeveloped world and it will increase interest rates in the developed world and decrease them in the undeveloped world. Such mobility is “generally good for financial asset-holders in the developed world and bad for those in the developing world; it is good for multinational corporations; and it is bad for (non-multinational) specific factors in the developed world and good for those in the developing world.” Frieden then goes on to analyze political economic interests given high levels of capital mobility. He does this by examining distributional effects. As noted before, high capital mobility forces countries into a trade-off between exchange rate and economic policy autonomy. Frieden now characterizes this as a trade-off between exchange rate and degree of exchange rate flexibility. He finds that international investors and traders are interested in giving up autonomy over fiscal policy in exchange for stability in the exchange rate. This is because a changing exchange rate leads to increased uncertainty for international trade. These same actors are most likely, he argues, to support international economic policy coordination. On the other hand, producers of non-tradable goods and those who produce for the domestic market are more interested in autonomy of fiscal policy than the exchange rate. This is because the exchange rate has little effect on their business and a fluctuating exchange rate may even help to keep out foreign competition.

Peter Katzenstein, Small States in World Markets: Industrial Policy in Europe (Ithaca, NY: Cornell University Press, 1985), chapters 1 (pp. 17-38) and 5 (pp. 191-211)

Small states in Europe could not brace themselves in the choppy waters of the international economy in the ways that large states did (in the US, protectionism; in Japan, industrial policy). Instead, they adopted a system of democratic corporatism, in which there is an ideology of social partnership expressed at the national level; a relatively centralized and concentrated system of interest groups; and voluntary and informal coordination of conflicting objectives through continuous political bargaining between interest groups, state bureaucracies, and political parties. Democratic corporatism does not magically transform social and political hostility into harmony; instead it offers an institutional mechanism for mobilizing the consensus necessary to live with the costs of rapid economic change. The small European states combine international liberalism with domestic compensation (rather than with protectionism, as in US, or structural transformation, as in Japan). The system arose in the 1930s and 1940s, during the Great Depression, Fascism and WWII. Every sector was vulnerable to shifts in the international economy, and thus external threats impressed upon the elites the need for internal unity and cooperation. The sense of crisis that originally led to economic cooperation between labor and business persists today, and it holds the system together: because the small European states are very open economies, political actors rarely lose the sense of being exposed to an international economy beyond their control. Increased economic adversity should not tear the democratic corporatist state apart, but rather reinforce it–a growing sense of vulnerability should, on questions of strategy in the international economy, unite political opponents in the small states who disagree on other substantive issues. Note that the small European countries are better off than developing countries in the Third World. Developing countries depend on the import of goods, capital, and technology, while the small European countries depend on exports. Developing countries are more vulnerable to declining terms of trade. Although the small European countries are dependent on international investment, they are better able to impose upon the MNCs standards of behavior compatible with domestic political goals.

Geoffrey Garrett, “Global Markets and National Politics: Collision Course or Virtuous Circle?” International Organization, 52, 1998, pp. 787-824

Garrett addresses two main questions in this paper. First, does increased market integration exert lowest-common-denominator pressures on national economic policies? (Garrett thinks not; it.s true that mobile capital has exit options, but domestic political opinion will encourage governments to enact redistributive policies, blunting the impact of integration.) Second, if governments are more activist in the economy (that is, if policy regimes do not converge around a free market ideal type), does this have macroeconomic costs? (Garrett thinks that costs can be balanced, and even benefits are possible, because MNCs can benefit from government spending and creation of high human and physical capital stocks.) After WW2, Bretton Woods facilitated trade liberalization and domestic compensation by combining fixed exchange rates with capital controls, thus leaving governments with monetary policy autonomy and allowing them to pursue Keynesian policies. After Bretton Woods was abandoned, Garrett argues that the wealthy OECD countries were able to maintain control over their domestic economies because, in the age of capital liberalization, they could (and do) let their currencies float. Mundell-Flemming problem solved (Mundell-Flemming: a country cannot have a fixed exchange rate, free movement of capital, and monetary policy autonomy; only two out of three is possible). Poor countries, however, tend not to have such stable currencies, as their domestic politics are generally more messy than those of the OECD. Forced to fix their currencies, then, countries integrated into the periphery of the international market are severely constrained in their ability to protect their populations from jarring economic change. They do not enjoy the tools of monetary policy available to OECD countries. Demonstrating that markets do not completely dominate domestic policy, Garrett shows that strong left-labor regimes responded to financial market integration with everhigher levels of public spending; whereas governments in countries with much weaker left parties and trade unions cut back the public economy. In short, the OECD countries have not converged around a less interventionist maroeconomic policy regime.

Torben Iversen and Thomas R. Cusack, “The Causes of Welfare State Expansion: Deindustrialization or Globalization?” World Politics, 52 (April 2000), pp. 313-349

Iverson and Cusack argue that the main driver of structural change in the economies of advanced industrial countries is not globalization, but rather domestic deindustrialization. And this deindustrialization, they write, is also a domesticallydetermined process, caused by shifting consumer preferences, from manufactured goods to services, as consumer wealth increases. Two cross-section time-series regressions constitute the majority of their evidence for these claims. The first shows that deindustrialization is highly correlated with increased government spending; the second indicates that domestic economic factors - productivity growth, income and capital formation–are correlated with deindustrialization, while various measures of trade balance and capital openness are not. So the theory is that rising wealth leads to consumers who spend an increasingly greater portion of their incomes on services, while their demand for manufactures eventually hits an upper bound and then stalls. This leads to a small portion of the workforce in manufacturing both directly, as a smaller portion of national income is spent on manufactured products, and indirectly: productivity in manufactures rises to squeeze profits from inelastic demand, which causes greater unemployment in industry. Thus we have domestically-driven deindustrialization–and Iverson and Cusack show that the more industry there was to begin with (in 1962), the more deindustrialization that has occurred in subsequent decades. To deal with this structural change, governments have adopted one of three strategies:

  1. US, UK, Canada, Netherlands: promote employment in privates services, often by deregulating product and labor markets and allowing greater wage dispersion; use various forms of public insurance to compensate workers for the risks of having to find new jobs in services.
  2. Scandinavia: state maintains extensive regulation of private services and relatively compressed wage structure while expanding employment in public services and expanding public insurance functions.
  3. Germany, France: maintenance of heavy regulation of labor and product markets; no expansion of public sector; welfare state effort geared towards promoting early retirement to deal with unemployment problem.
    In brief, growth in both transfers and government consumption can largely be explained as a function of the severity of internally-driven employment losses in the traditional sectors, not by forces in the global economy. The partisan orientation of government matters too, though, with leftist-labor parties enacting more redistribution.

Robert R. Kaufman and Alex Segura-Ubiergo, “Globalization, Domestic Politics, and Social Spending in Latin America: A Time-Series Cross-Section Analysis, 1973-97,” World Politics 53 (July 2001), pp. 553-587

Using time-series cross-national regression models to analyze the effects of trade and capital integration on welfare spending in Latin America, Kaufman and Segura-Ubiergo conclude that, “the overwhelming weight of evidence favors the efficiency over the compensation hypothesis” (582). That is, trade integration has tended to reduce social expenditures in Latin America rather than “compensate” the losers of economic change by expanding social safety nets. Kaufman and Segura-Ubiergo emphasize how this is a function of domestic political factors. Social security transfers benefit mainly formal-sector, unionized labor, which tends to be a relatively small group, and, as a consequence of liberalization, an increasingly powerless one. Therefore only leaders who are dependent on labor support tend to protect pensions and other transfers to formal-sector workers; others can afford, politically-speaking, to cut these benefits, as protest is expected to be relatively weak, and cut they do. The relative political ease of cutting pensions highlights the fact that, even in the more labor-leaning governments, social protections are not extended to much of the population, namely farmers and informal-sector workers. Protections from market forces are already weak in Latin America, and trade integration seems to make them only weaker. One note of optimism presented in this study is that capital integration tends to increase state spending on health and education (though trade integration does not). Though capital liberalization compounds the tendency to reduce social safety nets, it does appear to encourage state-supported development of human capital. Note: Efficiency Hypothesis: high levels of social spending reduce competitiveness in global markets; therefore, more international competition leads to businesses pressing governments to reduce social expenditures. Compensation Hypothesis: the welfare state is a mechanism to offset social costs of international integration and will contribute to the development of human capital; to the extent that public investment in human capital provides a collective good for the private sector, business groups might welcome or even press for these expenditures.

Judith Goldstein, “International Law and Domestic Institutions: Reconciling North American ‘Unfair’ Trade Laws,” International Organization 50 no. 4 (Autumn 1996), pp. 541-564

Nations agree to specific rules of international conduct when these rules present a solution to a domestic problem; in other words, although international agreements are a constraint on domestic policy, these constraints are accepted because they provide opportunities for domestic actors to further their own interests. Goldstein presents the case of the FTA and NAFTA, highlighting how these trade agreements furthered the interests of the US president against his own bureaucracy. American producers can petition US trade boards to add tariffs to imported goods if foreign companies are “dumping” the goods on the American market (or just producing them more cheaply than American firms do). The president might wish to liberalize trade, but he has no control over what the trade boards do. NAFTA (and FTA before that) provided a solution : domestic policy regarding US trade with Canada would be constrained, but since it wasn.t under the domain of the president anyway, it was hardly a diminution of executive power; and trade liberalization, which is what the president, and even most of the Congress wanted, was achieved.

Democratization and Regime Change

Concept Map

The literature on regimes and regime change deal with the questions: how do we define democracy? What are the conditions that enable countries to transition to democracy? What prevents the transition from authoritarian to democratic regimes in other countries? Why are some countries able to sustain democracy while others cannot? While some of the works below aim for a universal theory of democracy that can apply to all times and places, others focus on middle-range theories limited to particular regimes and times. Portions of these notes draw from the Harvard Department of Government Comparative Politics Field Seminar summaries of 1998 and 2000.

Robert Dahl, Polyarchy: Participation and Opposition (New Haven: Yale University Press, 1971), ch 1 (pp. 1-16), chapters 3-6 (pp. 33-104)

Dahl defines democracy as a regime in which government is responsive to citizens, and all full citizens have unimpaired opportunities to formulate preferences, to signify preferences to their fellow citizens and the government by individual and collective action, and to have their preferences weighed equally in the conduct of the government. Since democracy is not likely to exist according to this definition, Dahl uses the term polyarchy. Polyarchy has two dimensions, public contestation (liberalization) and the right to participate (inclusiveness). High inclusiveness and high liberalization is polyarchy, high inclusive and low liberalization is inclusive hegemony, low inclusiveness and high liberalization is competitive oligarchy, and low inclusiveness and /low liberalization is closed hegemony. There are three paths to democratization: (I) liberalize, then expand inclusiveness, (II) Expand inclusiveness, then liberalize, and (III) liberalize and expand inclusiveness at the same time. England and Sweden followed path I. Germany followed path II. France followed path III. The first path is most favorable because in the other two paths a greater number of interests are involved before or at the same time as the introduction of contestation, which makes it hard to establish consensus. Although the first path is preferable, it is not likely to be a viable path for most regimes because most regimes have already extended the franchise, i.e., inclusiveness is already high. Risks of failure of democratization can be reduced if steps toward liberalization are accompanied by a dedicated search for a system of mutual guarantees. Dahl then discusses the various types of transition or implementation. The likelihood that a government will tolerate opposition increases as the resources available to the government for suppression decline relative to the resources of an opposition. The likelihood that a government will tolerate opposition increases with a reduction in the capacity of the government to use violence or socioeconomic sanctions to suppress an opposition. Two general kinds of circumstances can reduce the capacity of the government to use violence or socioeconomic sanctions against an opposition: (1) these factors sometimes cease to be available as political resources, and (2) these (and other) political resources may be so widely dispersed that no unified group has a monopoly over them. The beliefs of the military and its tendency to intervene in politics are important (Chile vs. Argentina). Competitive politics is most likely when access to violence and socioeconomic sanctions is either dispersed or denied to both the opposition and the government. Agrarian societies have generally fallen into either traditional peasant societies (high inequality) or free farmer societies (more egalitarian). Three factors are beliefs about equality, the degree of equality in the distribution of land, and the state of military technology. In the traditional peasant society, these three factors all operate in the same direction-cumulative inequalities of status, wealth, income, and means of coercion mean a marked inequality in political resources, which is reinforced by prevailing beliefs. This leads to inequalities in political resources and thus hegemonic political systems. The free farmer society is the opposite, leading to a pluralistic social order.

Seymour Martin Lipset, Political Man: The Social Bases of Politics (Garden City, NY: Doubleday, 1960), chapter 2

“Men may question whether any aspect of this interrelated cluster of economic development, Protestantism, monarchy, gradual political change, legitimacy, and democracy is primary, but the fact remains that the cluster does hang together” (57). Lipset considers two characteristics of a society “which bear heavily on the problem of stable democracy: economic development and legitimacy, or the degree to which institutions are valued for themselves and considered right ad proper” (28). Lipset wants to test the common hypothesis that democracy is related to economic development: “the more well-to-do a nation, the greater the chances that it will sustain democracy” (31). Lipset finds that “In each case, the average wealth, degree of industrialization and urbanization, and level of education is much higher for the more democratic countries”(33). Education matters: “If we cannot say that a ‘high’ level of education is a sufficient condition for democracy, the available evidence suggests that it comes close to being a necessary one” (40).In Western non-democratic states, national economic development (per capita income) is more strongly correlated with the strength of Communists and other “extremist groups” than ethnic or religious factors. (46) But Lipset acknowledges that economic hardship or poverty per se may not necessarily be the main cause of radicalism. Creating a national culture as opposed to a distinct lower-class one: “Increased wealth and education also serve democracy by increasing lower classes’ exposure to crosspressures which reduce their commitment to given ideologies and make them less receptive to extremist ones” (50). The general income level matters because it makes losing more palatable: “If there is enough wealth in the country so that it does not make too much difference whether some redistribution takes place, it is easier to accept the idea that it does not matter greatly which side is in power. But if loss of office means serious losses for major power groups, they will seek to retain or secure office by any means available” (51). Conversely, “The poorer the country, the greater the emphasis on nepotism,” which in turn “reduces the opportunity to develop the efficient bureaucracy which a modern democratic state requires” (52).

Adam Przeworski and Fernando Limongi, “Modernization: Theories and Facts,” World Politics, 49 (January 1997), pp. 155-183

In short, Przeworski and Limongi find that democracies are increasingly stable as per capita incomes rise. This differential stability accounts for the oft-noted correlation between democracy and wealth. There are no grounds to believe that economic development breeds democracies; rather, democracies are born for other reasons, and just happen to survive better in wealthier countries than in poorer ones. The relationship between levels of development and the incidence of democratic regimes is strong. There are two main lines of argument for why this may be the case: (1) the endogenous argument posits that democracies may be more likely to emerge as countries develop economically, or (2) the exogenous arguments posists that democracies may be established independently of economic development but be more likely to survive in developed countries. The endogenous theory is associated with modernization literature, and considers the complexification of society at the time of economic development, a process which allegedly makes running a society by command increasingly difficult. The exogenous theory may be supported by the observation that old dictatorships seem to have fallen and been replaced by democracies because of failure in war, the death of a founder, economic crises, etc: in other words, causes not associated with issues of economic development. Przeworski and Limongi sympathize with exogeneity theorists, saying that the level of economic development per se only effects the chances of survival for an already-existing democracy established by political actors. This article has been sharply criticized, particularly in Boix and Stokes’ (2003) “Endogenous Democratization” World Politics. Boix and Stokes replicate the analysis using data that includes pre-1950s cases of democratization, arguing that the focus on post WWII cases biased Pzeworski and Limongi’s conclusions. Using their dataset, Boix and Stokes show that development was strongly associated with development in the First and Second Wave of democratization. Despite their criticism of the exogenous democratization argument, Boix and Stokes do find evidence for it, even controlling for variables that they claim Pzeworski and Limongi omitted like educational attainment and income inequality. In short, they suggest both an endogenous and exogenous impact on democratization.

Dankwart Rustow, “Transitions to Democracy: Toward a Dynamic Model,” Comparative Politics, 2 (April 1970), pp. 337-364.

Rustow distinguishes between reasons for why democracy arises than for why democracy remains stable. This distinction goes unrecognized in studies that use correlational data to establish causation or studies that keep political factors as solely dependent variables produced by social and economic independent variables. Rustow posits that democratization need not be continuous, linear, or homogeneous in geography, time, or social attitudes. Excluding democratization that was externally imposed (either by military force or immigration), Rustow proposes a model for democratization that begins with national unity, that is, citizens accept that they are members of the same polity. Then, there is a preparatory phase with prolonged and inconclusive political struggle, that can, but need not, be about economic issues. The issue becomes polarized, but not along regional lines that would allow for secession. In addition, widespread values must prevent the conflict from degenerating into a genocidal attack or attempt to expel a population. The third phase is the decision phase where elites decide to compromise, attempting to reconcile unity with diversity. The decision is made by an oligarchy, perhaps with influence by splinter groups or temporary new actors. The decision to use democratic procedures to compromise can be incidental to the substantive issues. After the decision, the factions will likely continue to disagree, both on ideal governmental structures and on substantive policy issues. The final phase is the habituation phases where elites have to transmit the compromise they have struck to the rest of society and train society to accept it. The example of the initial compromise helps to make later compromises appear achievable, and as old leaders die off or retire, the new leaders will embrace democracy wholeheartedly. Parties work to expand the electorate, much as they do in Huntington’s formulation. Failure to compromise on key issue during the decision phase can doom the transition during the habituation phase. Rustow notes that compromise is easier on distributive issues, in which the parties can split the difference, than it is on community issues, such as ethnic or religious identity. Rustow concludes that this model presents the necessary conditions for democratic transition, and that they must occur separately and sequentially. Other proposed initial conditions, such as wealth or education, are simply products of the habituation phase. The essence of democracy is dissension and conciliation, which the model presents.

Carles Boix, Democracy and Redistribution (Cambridge: Cambridge University Press, 2003), pp. 1-59

The central premise of Boix’s theory is that political regimes are systems that mediate pressures for redistribution between society’s economic classes. He identifies three causative factors that determine whether regime transitions occur: inequality, capital mobility, and lower-class political mobilization. (1) If the wealthy constitute the principal interest-group supporting an authoritarian regime, and society has a highly unequal distribution of wealth, democracy leads to the empowerment of the poor and the subsequent implementation of heavy redistribution programs. The wealthy therefore have sizeable incentives to oppose democracy. Alternatively, low inequality lowers the costs of redistribution, and encourages voluntary democratization by the wealthy. (2) If the wealthy have immobile capital assets which are susceptible to expropriation and redistribution (like farms, which cannot be moved to another country or protected from taxes or expropriation), they will have more to lose from democracy, and will oppose it. Mobile assets (such as human capital or financial investments) can be sheltered from redistribution. In this case, Boix asserts that pressure against democratization will subside, and the elite will democratize. On face these seem to be merely permissive conditions for democratization, because it is not clear where “demand” for democratization comes from. In other words, even if democracy is “cheaper” for the wealthy elite, it is unclear why they would purposefully and voluntarily give up power. It is here that Boix’’s description of lower-class political mobilization is crucial. Boix asserts that when the lower class mobilizes and overcomes its collective action problem, the elite faces a choice of whether or not to repress. This choice is evaluated in the context of the costs of repression (in the face of an organized lower-class) versus the costs of democratization outlined above. Therefore, the organizational and political capabilities of both parties (the elite and the poor) matter. It is unclear, however, what the source of these organizational capabilities is, or why the poor may overcome their collective action problem. Crucially, political mobilization by the lower-class in highly unequal societies may lead to war instead of a negotiated solution, because the parties are unclear about their mutual capabilities, and may fight rather than risking losing out in a bargaining process (Fearon).

Daron Acemoglu and James A. Robinson, Economic Origins of Dictatorship and Democracy (Cambridge: Cambridge University Press, 2006), chapters 2-3 (pp. 15-87)

Acemoglu and Robinson’s theory of democratization is fundamentally economic and conflictual. Most policy choices have economic consequences creating clear winners and losers. Politics is inherently a zero-sum game pitting elites against citizens. Elites, who oppose taxation and redistribution, are the clear winners in dictatorships, which tend to enact policies that match their preferences. Citizens, on the other hand, prefer democracy because of their interest in taxation and redistribution. Central to Acemoglu and Robinson’s argument is the distinction between de facto and de jure political power. In dictatorships, elites hold de jure power–actual control of the levers of government–while citizens often possess de facto power because of their superior numbers. Conversely, de jure power belongs to citizens in democracy, but in times of political upheaval elites may obtain de facto power by allying with the military. Real political power consists of both de facto and de jure power. This distinction becomes important in Acemoglu and Robinson’s story of democratic transition and consolidation. Consider a dictatorship in the throes of political instability, facing what AR term “a revolutionary threat.” Citizens may possess de facto power because of their numerical ability to overwhelm the elites, but they recognize that power of this sort is transitory, they are powerful today, but may not be powerful tomorrow. “The citizens would like to lock in the political power they have today by changing political institutions–specifically, by introducing democracy and greater representation for themselves–because without the institutional changes, their power today is unlikely to persist” (25). In other words, they desire de jure power. Facing a potential revolution–for them, the worst possible outcome–elites are inclined to give citizens what they want today: redistribution and other pro-majority policies. Crucially, citizens recognize that pro-majority policies today do not necessarily entail pro-majority policies tomorrow. Democratization is the only credible promise elites can make to enact pro-majority policies tomorrow. Of course, elites can also choose to repress citizens, or they can renege on their commitment to pro-majority policies after the establishment of a democracy. Democratization and consolidation are influenced by various comparative statics. Inequality plays a prominent role in these comparative statics–highly equal societies are unlikely to democratize because there are limited redistributive gains from democratization. States experiencing medium levels of inequality likely to democratize and consolidate because the poor demand it and the elites have less to lose, and in turn less incentive to repress. Societies with high levels of inequality are unlikely to democratize because the elites have much to lose and prefer repression.

Ziblatt, Daniel, “How Did Europe Democratize?” World Politics 58:2 (January 2006), 311-38.

This article reviews Boix, Acemoglu and Robinson, Collier, and Tilly’s books on democratization, which “make it clear that the advent of democracy in Europe in the late 19th century was not the exceptional and overdetermined outgrowth of modernization as traditionally portrayed by comparative scholarship” (311). Democracy emerged historically as a process of discrete institutional reforms that, at times, undercut each other.

Juan Linz, “The Perils of Presidentialism,” Journal of Democracy 1, No. 1 (Winter 1990)

Linz argues that parliamentary systems are better for the consolidation of democracy and should replace presidential systems. He makes three main theoretical arguments:

  1. Presidentialism is less flexible than parliamentarism. Unlike in a parliamentary system where the party who is voted into power, in a presidential system an individual is elected for a fixed term. Since no easy mechanisms exists to replace presidents who have lost the confidence of the legislature, changes in government leadership are difficult and can easily lead into unstable political (and thus economic and social) situations. In parliamentary systems on the other hand, leadership changes are readily facilitated and can be done with a minimum of governmental disruption. By replacing one leader for another, the majority party stays in power and party policies can be kept intact. A system based on majority party rule is more flexible to change than a system based on the rule of an individual.
  2. Linz contends that presidentialism is more prone to immobility and gridlock than a parliamentary system. The separation of powers between the executive and the legislative, both of which are elected by the citizens, contributes to political conflict and instability. In political confrontations, each branch can claim to represent the people, creating dual-legitimacy problems that can lead to gridlock and a political stalemate. This conflict can also lead to polarization, policy rigidity and conflict as parties attempt to differentiate themselves and create strongholds of support. Legislative paralysis and confrontation between congress and the president undermine regime stability. In contrast, the parliamentary system eliminates the problem of dual legitimacy and promotes effective government by curtailing stalemates between the executive and legislative branches.
  3. Presidentialism is not conducive to developing or consolidating democracy because the victor alone controls the executive. This argument follows from the logic that the political party is an essential institution for political representation in a modern democracy. The partisan conflict that develops from the “winner-takes-all logic” of presidential systems can lead to fractionalization and instability, undermining the system of political parties. A parliamentary system on the other hand, curbs partisan conflicts and promotes the development of cohesive and disciplined parties.

Huntington, Samuel P., The Third Wave: Democratization in the Late Twentieth Century (Norman: University of Oklahoma Press, 1991), pp. 13-108

This book is an attempt to explain and analyze the transition of approximately 30 countries from nondemocratic to democratic systems between 1974 and 1990. It is neither a work of history or theory; to the extent that it does generalize, its generalizations are limited to the events of the 1970s and 1980s that it discusses. Huntington favors a procedural definition of democracy, classifying a system “as democratic to the extent that its most powerful collective decision makers are selected through fair, honest, and periodic elections in which candidates freely compete for votes and in which virtually all the adult population is eligible to vote” (7). The two dimensions he highlights, referencing Dahl’s work on polyarchy, are contestation and participation. He suggests that these dimensions imply basic civil and political freedoms. Huntington’s concept of a “wave” of democratization refers to “a group of transitions from nondemocratic to democratic regimes that occur within a specified period of time and that significantly outnumber transitions in the opposite direction during that period of time” (15). During a wave of democratization, some regimes are also likely to liberalize but not fully democratize. The 3rd wave began in 1974 with the end of the Portuguese dictatorship. Over the course of the third wave, about 30 countries replaced authoritarian regimes with democratic ones. The wave began in southern Europe, moved to Latin America and Asia in the late 1970s, and finished in the communist world in the late 80s. Huntington proposes that no single factor is necessary or sufficient for democratization. In each country, a combination of causes is at work which is likely to vary from country to country. In addition, within a single wave, early regime changes are likely to have different causes than latter ones. Given this, however, the most significant explanatory variable in democratization is the behavior of political elites. The first wave was primarily caused by economic and social development, and by Allied victory in World War I. The second wave was mostly caused by political and military factors, to include the victory of the Western allies in World War II and decolonization. Huntington identifies five causes that seemed to be significant in bringing about the 3rd wave: 1) A deepening legitimacy problem in authoritarian regimes caused by a “world democratic ethos” and aggravated by military defeats, economic problems, and oil shocks. Authoritarian regimes have a special “performance dilemma”: unlike democracies, where legitimacy comes from procedures for selecting governments, authoritarian regimes must often base their legitimacy on their performance (50). 2) Global economic growth in the 1960s. However, Huntington emphasizes that “economic factors have significant impact on democratization but they are not determinative” (59). 3) Liberalization of the Catholic Church (75% of 3rd Wave countries were Catholic). After the mid-1960’s, the Church almost invariably opposed authoritarian regimes. 4) Changes in the policies of external actors, mainly the EC, US, and USSR: In the late 1960s, the EC changed its attitude toward expanding its membership; the US shifted its foreign policy beginning in 1974 away from Realpolitik and toward the promotion of human rights and democracy in other countries. 5) “Snowballing” or demonstration effects. Early regime transitions stimulated subsequent efforts in other countries. Earlier transitions showed that democratization could be brought about, and how it could be done.

Barbara Geddes, “What Do We Know about Democratization after Twenty Years?” Annual Review of Political Science 2 (1999): 115-144.

Reviews a large number of studies about late 20th century democratization and finds to claims to receive strong support: (1) democracy is more likely in more developed countries, and (2) regime transitions of all kinds are more likely during economic downturns. Looking at a single regime characteristic, leadership, Geddes distinguishes between military, personalist and single-party regimes. Using game theoretic models, Geddes puts forth the notion that these three types of regimes tend to break down in different ways because of the distinct incentives faced by their cadres. Military regimes (Battle of the Sexes) tend to split when challenged because, according to Geddes, “most professional soldiers place a higher value on the survival and efficacy of the military itself than on anything else.” When factional splits inevitably arise in the course of military rule, a return to the barracks becomes an attractive option to most officers. Thus, military regimes tend to break apart more quickly than other regimes. They are also most susceptible to economic down turns and are most likely to negotiate their transfer of power to a civilian government, often leading to democracy. Personalist regimes (pseudo- Staghunt game) are those regimes in which a single leader dominates the military and state apparatus, and the ruling party if there is one. They tend to last longer than military regimes (they are less vulnerable to splits), but not as long as single-party regimes (because of succession issues, narrow support bases, and a particular vulnerability to economic downturn due to a reliance on patronage). In times of crisis, such regimes tend to “circle the wagons,” to use Geddes’ phrase. The preferences of single-party cadres (Staghunt) are much simpler than those of officers: they simply want to remain in office. With no other “day job” to return to, individual cadres have no incentive to do anything but cooperate with each other. In case of factional splits, most cadres will keep their heads down until leadership struggles have been settled. Cooptation, rather than exclusion, is usually the rule. Single-party regimes have the fewest endogenous sources of instability and therefore last longer than their military or personalist counterparts.

Michael Ross, “Does Oil Hinder Democracy?” World Politics 53, No. 3 (2001): 325-361

Ross tests a number of claims regarding the widely-held hypothesis that oil-impedes democracy. First, is it true? (yes); Second, is it generalizable outside of the Middle East region and to other natural resources and commodities such as minerals (e.g. diamonds, etc.)? (yes); And, third, what are the causal mechanisms through which oil wealth is thought to impede democracy? He tests three mechanisms found within the literature. Among these are variations on the “rentier effect,” in which oil-rich states seek rents from oil (1) at the expense of broader development initiatives and can keep taxes low, (2) pay off potential opponents with their wealth, and (3) prevent the formation of social groups independent from the state in order to resist democratic pressures. The second mechanism is the “repression effect,” in which oil states can spend large sums of money on a repressive apparatus that represses democratic dissenters and opponents of the regime. The third dominant mechanism through which oil-rich states are thought to avoid democratization is the “modernization effect.” This causal mechanism assumes that the democratizing effects of capitalism are associated with its tendency toward the differentiation of the labor force, urbanization of the population, and increased public services such as education and health care. However, in oil economies, these processes are retarded so that wealth grows without a corresponding increase in democratic pressure. Ross concludes that the results of these tests are “at least weakly consistent with each of these causal mechanisms.” Particularly, 1) the results are consistent with all three aspects of the rentier effect; 2) that oil wealth “may be linked to higher levels of military spending, which in turn tends to impede democracy” but not to numbers of military personnel; and 3) that occupational specialization is significantly associated with democracy while education, life expectancy, and other proxies for “modernization” are not.

Steven Levitsky and Lucan A. Way, Competitive Authoritarianism: Hybrid Regimes after the Cold War (Cambridge University Press, 2010)

Competitive authoritarianism: a post-Cold War phenomenon in which regimes combine electoral competition with varying degrees of autocracy. Competition is real, but unfair. “Competitive authoritarian regimes are civilian regimes in which formal democratic institutions exist and are widely viewed as the primary means of gaining power, but in which incumbents’ abuse of the state places them at a significant advantage vis-a-vis their opponents” (4). Distinguishing competitive authoritarianism from democracy is a violation of at least one of the follow attributes of democracy: (1) free elections, (2) broad protection of civil liberties, and (3) a reasonably level playing field. The central argument focuses on two main factors: (1) ties to the West (linkage), and (2) the strength of the governing party and state organizations (leverage). Linkage serves as the transmitter of international influence and refers to the density of ties (economic, political, diplomatic, social, and organizational) and cross-border flows (of capital, goods, services, people, and information) between particular countries and the US, EU, and Western dominated multilateral institutions. The most important source of linkage is geographic proximity.Leverage is a governments’ vulnerability to external democratizing pressure and is rooted in three factors: Western foreign policy objectives, the strength of the country’s state and economy, and the role of counter-hegemonic powers (Russia, China, Japan, and France). How do we get competitive authoritarianism? Autocrats adopted formal institutions because they were incentivized to do so by the new world system: they needed to compete for scarce international resources (no more bi-polar system), there was a shift in Western foreign policy to some semblance of democratic conditionality (we support dictators slightly less), new international frameworks developed, and transnational organizations emerged. In states with extensive ties to the West, post-Cold War international influences were so intense that they contributed to democratization even where domestic conditions were unfavorable. In these cases, the international variables are primary. Where linkage is high, leverage is more likely to generate pressure for full democratization. Where linkage is low but leverage is high (sub-Saharan Africa), international pressure may be significant, but tends to be limited and sporadic. Where ties to the West were less extensive, post-Cold War international pressure was weaker, and consequently, domestic factors weighed more heavily. Here, domestic structural variables (mostly the strength of the state and governing party organizations; i.e. the balance of power between autocrats and their opponents) are paramount.

Michael McFaul. “The Fourth Wave of Democracy and Dictatorship: Noncooperative Transitions in the Postcommunist World.” World Politics 54, no.2 (January 2002): 212-244

McFaul begins with the premise that the analytic framework received from scholarship on the third wave of democratisation does not explain the outcomes in the postcommunist world. In the third wave, an even balance of power between the old regime and and the democratic opposition predicted a “pacted” transition to democracy. McFaul argues that this pattern is not obvious in the postcommunist world, as most transitions did not produce democracy and even the successful democratisers did not follow the “pacted” path. On the contrary, it was situations of unequal distributions of power that produced the quickest and most stable transitions. The ideological orientation of the more powerful party largely determined the type of regime to emerge. McFaul outlines three alternative causal pathways in transitions from communism: 1) Imposition from below: hegemonic democrats where societal mobilisation was critical for producing democratic leaders, who in turn were important actors by virtue of their societal support. The new regime was produced by democrats with power, through revolutionary rather than evolutionary means. Examples: Poland, Hungary (limited elements of pacting), Czechoslovakia, Baltic States, East Germany (no pacting). 2) Imposition from above: hegemonic autocrats where the distribution of power remained in favour of the old regime. Democratic mass movements were either weak or absent, and leaders did not take a principled stand for democracy but rather consolidated their own power. Examples: Kazakhstan, Kyrgystan, Turkmenistan, Uzbekistan, Belarus. 3) Stalemated transitions: protracted confrontation and imposition resulting from from equal distributions of power. Outcomes: electoral democracy in Moldova and Mongolia, fragile and partial democracies in Russia and Ukraine, and civil war followed by autocracy in Tajikistan.

Wood examines the puzzle of why the elites of two oligarchic societies (El Salvador and South Africa) abandoned their implacable opposition to democratic rule and negotiated a transition to democracy with the insurgent counterelite. Wood finds that in South Africa and El Salvador, sustained mobilization by poor and working-class people transformed key interests of economic elites, leading to pressure on the state to compromise with the insurgents, thereby strengthening regime moderates over hard-liners with the result that negotiated transitions to democracy followed. Wood calls this the “insurgent path to democracy.” In El Salvador, (rural) insurgency, together with the state’s counterinsurgency policies, had two economic effects: 1) rapid decline in national output and 2) a sectoral shift from traditional export agricultural production to commercial and service sectors. With its diversified economic interests, the economic elite became more tolerant of democratic norms and aspiration. [There was a massive sectoral shift in the core interests of the economic elite.] Woods argue that it quite doubtful that a transition to democracy would have occured as it did without the shift in elite economic interests. In South Africa, private investments fell as unrest increased. (Unionists succeeded in buidling various unions and federations in the 1970s and political mobilization in townships also surged.) Wood argues that sustained insurgency had an effect on short- and long-term praivate capital flows to South Africa and shaped the movement of other economic variables such as the exchange rate and the cost of capital. Political mobilization posed a general threat to elite economic interests because major business agglomerations in South Africa were highly diversified and business leaders individually held a wide profolio of interests. In her interviews, business elites emphasized labor mobilization as central to their move to support political reform (toward a liberal nonracial democracy).

Dan Slater, “Revolutions, Crackdowns, and Quiescence: Communal Elites and Democratic Mobilization in Southeast Asia,” American Journal of Sociology 115:1 (July 2009)

Slater asks why do opposition groups mobilize in some settings but not others? Slater’s key idea is that a communal elite, meaning a society’s primary possessor of nationalist or religious authority, is the pivotal player in sparking democratic mobilization. When autonomous communal elites exist, they can lead democratic mobilizations at times of crises. Variation in the existence of communal elites can explain variation in the occurrence of democratic mobilization in similar conditions of economic crisis, oppression, and so on. Communal elites use their symbolic power (often in the form of nationalism or religion, though this is never really defined) to overcome the rational choice puzzle of why individuals participate in high-risk events. In short, there are structural factors–based on the presence or absence of communal elites–that shape whether opposition emerges. These factors are separate from whether opposition succeeds. Communal elites gain autonomy and political salience in three ways: 1) dynastic rulers: elites who gain power through high colonialism and are not destroyed by colonial rule, 2) religious notables: hegemonic national religion gives power to religious figures, 3) national leaders: gain power through decolonization process and maintain autonomy through onset of authoritarianism. Where a country had an independent set of these types of communal elites, protest was more likely.

Michael Bratton and Nicolas Van de Walle, “Neopatrimonial Regimes and Political Transitions in Africa,” World Politics 46, No. 4 (July 1994), pp. 453-489

The authors argue against the views that political transitions are driven by the initiative of key actors, and against a focus on deep social and economic structures. Rather, institutional characteristics of the preexisting political regime impart structure to the dynamics, and to a lesser extend the outcomes, of political transitions. The distinctive institutional hallmark of African regimes is neopatrimonialism. In neopatrimonial regimes, the right to rule is ascribed to a person rather than an office, and chief executive retains authority through personal patronage, rather than through ideology or law. Neopatrimonial regimes depart from the path followed by democratizing regime in Europe and Latin America. Political transitions from neopatrimonial regimes originate in social protest (rather than the corridors of elite power). Neopatrimonial elites fracture over access to patronage (the distinction between hard and soft liners does not capture this essential fault line). Elite pacts are unlikely in neopatrimonial regimes. Incumbent and opposition usual polarized, and political leaders may represent only a tiny coterie of clients. In neopatrimonial regimes, political transitions are struggles to establish legal rules (whereas corporatist regimes are already governed by rule of law). During transitions from neopatrimonial rule, middle-class elements align with the opposition (whereas corporatist regimes draw support from the expanding middle classes).

Civil Society and Social Movements

Concept Map

The question of whether human beings are naturally social underlines the literature on civil soceity and social movements. Some authors (Olson, Putnam) view collective action as a problem that must be overcome through incentives or social capital. Others suggest that poeple act collectively all the time (Tarrow, McCarthy and Zald, Kitschelt, O’Brien). A second question that animates this literature, and which ties this literature to the works on the State and Regimes, is what is the relationship of groups to institutions of the state and regimes.

Mancur Olson, The Logic of Collective Action: Public Goods and the Theory of Groups (Harvard University Press, 1965), pp. 1-52

Published in 1965, Olson’s book challenged the accepted wisdom in Olson’s day that out of self-interest, people will act collectively as a group to achieve their common interest. He focuses on groups that work to further the common interests of their members, i.e. labor unions, farm organizations, cartels, corporations and states. He argues that: 1) Group action does not follow logically from the premise of rational and selfinterested behavior. 2) Unless the number of people in a group is small, or unless there is coercion, or unless selective incentives are offered individually on the condition of contribution, rational, self-interested individuals will not act to achieve their common or group interests. 3) Small groups and large groups are not only quantitatively but qualitative different. Large organizations have difficulty in getting members to contribute voluntarily. For example, the nation state, with all the emotional resources at its command, needs to levy taxes on its citizens. Small groups (especially those where group members have asymmetric interest in the collective good), can provide themselves with collective goods without relying on coercion or selective incentives, because at least one member finds that his personal gain from having the collective good exceeds the total cost of providing some amount of the collective good (P.34). Groups are either inclusive (larger) or exclusive (smaller) depending on the nature of their goals (P.39). Relationship among members in these two types of groups are quite different. Members in exclusive groups must be sensitive to other members in the group, therefore exclusive groups want to limit group size, and usually require 100 percent participation (i.e. oligopoly). There is less strategic interactions among members in inclusive groups. In “privileged” groups, at least one member will gain more from a collective good than it would cost him to provide it unilaterally. In such groups collective goods can be achieved without much coordination. In “intermediate” groups, a member withholding his contribution will cause a noticeable decrease in the supply of the good, the collective good may or may not be achieved, and some group organization is needed. In “latent” groups, one member withholding his contribution will NOT cause a noticeable decrease in the supply of the good, therefore selective incentives need to be offered to stimulate members to act in a “group-oriented” way.

John McCarthy and Mayer Zald, “Resource Mobilization and Social Movements: a Partial Theory” American Journal of Sociology 82 (1977), pp. 1212-41

Social movements take place when rational decision-makers mobilize their followers and promote their cause with the best available strategies given limited cognitive and material resources. The resource mobilization model explains how political entrepreneurs overcome the challenge of organization. Three main difference exist between traditional theories of social movements and the resource mobilization theory: 1) support base, 2) strategy and tactics, 3) relation to a larger society. Support Base: Traditional theories of social movements focus on the aggrieved population which provide the necessary resources and labor. External supports are not incorporated as central analytic components. The resource mobilization theory argues that social movements may or may not be based upon the grievances of the presumed beneficiaries. Conscience constituents, individual or organizational, may provide major sources of support. And in some cases supporters–those who provide money, facilities, and even labor–may have no commitment to the values that underlie specific movements. Strategy and tactics: Traditional theories argue that tactics depend upon prior history of relations with authorities, prior experience of encounters, and ideology. Resource mobilization theory argues that social movement organizers have a couple of tactic tasks: mobilizing supporters, transforming the mass and the elite into sympathizers, achieving change in targest. Dilemmas occur in the choices of tactics. Tactics are influenced by inter-organizational competition and cooperation. Relation to a larger society: Traditional theories treat society and culture as descriptive, historical context; they emphasize the effect of environment upon movement organizations, but ignore ways in which movements can utilize the environment for their own purpose. Resource mobilization theory points argues that the society provides the infrastructure which social movement industries utilize. Aspects utilized include communication media, expense, levels of affluence, degree of access to industrial centers, preexisting networks, and occupational structure and growth.

Sidney Tarrow, Power in Movement (Cambridge University Press, 1996), p. 9-27; 62-78

For Tarrow, the problem for social movements is not that of free riders (as suggested by Olson) but of coordination difficulties (in Tarrow’s words, the problem is not individual, but social). Tarrow’s main argument is: People join in social movements in response to political opportunities, and then, through collective action, create new ones. Movements do not appear in direct response to the level of supporters’ grievances. It is the political opportunity that turns potential for movement into actual mobilization (18). The most important opportunities are changes in the structure of political opportunity. “Political opportunity structure” is “consistent dimensions of the political environment which either encourage or discourage people from using collective action”(18). Political opportunities are usually resources external to the group that can be taken advantage of. The most salient changes in opportunity structure result from opening up of access to power, from shifts in ruling alignments, from the availability of influential allies and from cleavages within and among elites. By mounting collective actions, organizers become focal points that transform external opportunities, conventions and resources into movements. The major external resources are social networks, and the cultural and ideological symbols. “Together, opportunities, repertoires, networks, and frames are the materials for the construction of movement (17). Repertoires of contention, social networks and cultural frames loser the costs of bringing people together in collective action, creating a broader and more widely diffused dynamic of movement (23). In Chapter 4, Tarrow looks at the relationship between the state and social movements. He argues that although movements always conceive of themselves as opposed to the institutions, collective action inserts them into complex policy networks, and therefore within the reach of the state. Movements enunciate demands in terms of frames of meaning that are comprehensive to a wider society; they use forms of collective action drawn from a existing repertoire; and they develop types of organization which often mimic the organization they oppose (26).

Herbert Kitschelt, “Political Opportunity Structures and Political Protest,” British Journal of Political Science, 16 (1986), pp. 57-85

Kitschelt in this article looks at anti-nuclear protests in France, Sweden, the US and West Germany, each pursued a different strategy and had a different impact on overall energy policy. The variations are explained by the “political opportunity structure” in a given country. In other words, institutional constraints have effects on the mobilization of social movements. Political opportunity structure are comprised of specific configurations or resources, institutional arrangements, and historical precedents for social mobilization, which facilitate the development of protest movements in some instances and constrain the others. Kitschelt says what differs his approach from resource mobilization (McCarthy and Zald) is its focus on movement variations both in terms of mobilization and impact. (58). The combination of political “input” and “output” structures in a country put limits on policy innovation. Where openness is high and capacity strong (Sweden), innovation tend to be greater (64).

Kevin O’Brien, “Rightful Resistance,” World Politics 49 (1) (1996): 31-55

Rightful resistance is a form of popular contention that operates near the boundary of an authorized channel, employs the rhetoric and commitments of the powerful to curb political or economic power, hinges on location and exploiting divisions among the powerful, and entails innovative use of laws, policies, and other officially promoted values to defy political and economic elites who have failed to live up to them. (P.33) Unlike everyday resisters (shirkers), rightful resisters seek rather than avoid attention of elites. Rightful resistance is a product of state building and of political opportunities created by established ideologies and rules.

Robert Putnam, Making Democracy Work: Civic Traditions in Modern Italy (Princeton University Press, 1993) (Chapters 3, 4, and 6), pp. 63-120; 163-185

A natural experiment: in 1970, the highly centralized Italian regime devolved much authority to 15 newly-created regional governments with essentially identical institutions. The treatment: differences in political culture. Outcome: areas with more social capital enjoyed better institutional performance than areas lacking in social capital. Putnam argues that formal institutional design is not sufficient in explaining variations in government performance. The key to better governance in Northern Italy is the “civic tradition” that takes form in norms and reciprocity and networks of civic engagement (167). For Putnam, collective action hampers attempts to cooperate for mutual benefit in all societies, and third-party enforcement is an inadequate solution to this problem. Voluntary cooperation depends on social capital. Norms of generalized reciprocity, and networks of civic engagement encourage social trust and cooperation because the reduce incentives to defect, reduce uncertainty, and provide models for future cooperation. Trust itself is an emergent property of the social system, as much as a personal attribute. Individuals are able to be trusting because of the social norms and networks within which their actions are embedded (177).

Robert Putnam, “Bowling Alone: America’s Declining Social Capital,” Journal of Democracy (1995)

Putnam surveys the decline of social capital in the US since 1950. He has described the reduction in all the forms of in-person social intercourse upon which Americans used to found, educate, and enrich the fabric of their social lives. He believes this undermines the active civil engagement a strong democracy required from its citizens. Putnam discusses ways in which Americans have disengaged from political involvement including decreased voter turnout, public meeting attendance, serving on committees and working with political parties. Putnam also cites Americans’ growing distrust in their government. Putnam attributes the three decades’ decline of civic engagement in America to the movement of women into the labor force, increased mobility and suburbanization, demographic transformation, and the technological transformation of leisure. Small, dense networks that enrich generalized trust and reciprocity and foster political participation are either replaced by virtual societies or have simply vanished in social changes. Putnam suggests closer studies of which forms of associations can create the greatest social capital, how various aspects technology, changes in social equality, and public policy affect social capital. He closes by emphasizing the importance of discovering how the United States could reverse the trend of social capital decay.

Sheri Berman, “Civil Society and the Collapse of the Weimar Republic”, World Politics 49 (1997), pp. 401-439

Berman argues that social capital may also lead to undesirable outcomes if the political institution and democracy in a specific country is not strong enough and is therefore overpowered by the social capital groups. Berman suggests that “it was weak political institutionalization rather than a weak civil society that was Germany’s main problem during the Wihelmine and Weimar eras” (90). Because the political institutions were so weak people looked to other outlets. “Germans threw themselves into their clubs, voluntary associations, and professional organizations out of frustration with the failures of the national government and political parties, thereby helping to undermine the Weimar Republic and facilitate Hitler’s rise to power.” In this article about the fall of the Weimar Republic, Berman makes the claim that Hitler rose to power so quickly because he was able to mobilize the groups towards one common goal. Even though German society was, at the time, a “joining” society, these groups were fragmented and their members did not use the skills they learned in their club associations to better their society. They were very introverted in the Weimar Republic. Hitler was able to capitalize on this by uniting these highly bonded groups under the common cause of bringing Germany to the top of world politics.

Theda Skocpol, Marshall Ganz, and Ziad Munson, “A Nation of Organizers: the Institutional Origins of Civic Voluntarism in the United States,” American Political Science Review 94 (3) (September 2000), pp. 527-546

Theda Skocpol et al. argues against the “small-is-beautiful” school of civic virtue proposed by theorists of social capital in favor of the large-is-good argument that translocal organizations with “sizable membership, reasonably long duration, and fairly large territorial extent.” People in the first camp include Robert Putnam, who according to Skocpol downplay the role of the state in its role of shaping civil society. They clearly believe that social capital is essential for the functioning of a democratic state, but do not give the state enough credit in shaping civic virtue, nor does Putnam give enough credit to large trans-local organizations in the United States. Olson is another author who believes that small groups are better at furthering their collective interest than large groups. In contrast Skocpol and co-authors argue that American voluntary organizations developed in close relationship to the representative and federal institutions of the United States. Further, trans-local linkages of large groups help sustaining the functions of local chapters. Scholars who focus on different countries in the world have also come up with similar accounts. Herbert Kitschelt compares the anti-nuclear movements in four democracies, France, Sweden, the United States and West Germany, and argues that the patterns of social mobilization vary according to a country’s “political opportunity structure,” in other words, the institutional arrangements of a society. Kevin O’Brien also showed that popular resistance movements sometimes adapt their operation to official norms in order to maximally challenge the authority, hence the “rightful resistance.”

Lily Tsai, Solidary Groups, Informal Accountability, and Local Public Goods Provision in Rural China”, American Political Science Review 101, No.2 (May 2007), pp.355-372

In democracies, formal institutional pf accountability are response for promoting public goods provision. However, in rural China, formal rules cannot guarantee public goods such as paved roads and electricity from their local government. Instead, solidary groups, that are “collections of individuals engaged in mutually oriented activities who share a set of ethical standards and moral obligations” offer moral rewards as incentives for local officials to provide public goods and services responsively (12). Solidary groups share the same objectives as formal institutions, that is, public goods provision; when formal institutions are weak, they provide “informal institutions of accountability” that are not written but nevertheless ensure good governance. In Helmke and Levitsky’s article, solidary groups find their counterparts in Peru, where peasants formed “self-defense patrols” and “communal assemblies” to maintain order and provide public goods in the midst of a military conflict (Helmke and Levitsky, 15). Tsai further argues that, in order for solidary groups to hold local officials accountable (at least in the case of China), they must be “encompassing” as well as “embedding” (13). Encompassing groups have their doors open to everyone under the local government’s administration, while embedding groups incorporate local officials into the group as members. Examples of “working” solidary groups are village-wide clans, temples, community festival groups and lineages. Examples of “malfunctioning” solidary groups are Christian churches, because their lack of embeddedness in the local government: village officials are discouraged from participating in church activities and are therefore not subject to moral accountability within groups. Lineages only have a positive impact on public goods provision when they are village-wide; when there are multiple lineages in a village, they can hinder public goods provision. The key to successful accountability is the moral constraints bounding members of a group. Members of solidary groups share a common set of ethical standards and moral obligations to other members of the group; members who fail to demonstrate his “loyalty” will naturally be punished with social ostracisim.

Ethnicity, Civil War, and Political Violence

This literatures seeks to explain the construction and assertation of ethnic identities, as well as the dynamics of ethnicity and violence. Horowitz, Bates, Latin, Kalyvas, Beissinger, Varshney, and Posner offer alternative understandings of ethnicity as “primodial,” as an intentional “strategic” social construct, or as an “unintentional” construct. Some works see ethnicity as a driving force in political violence (Horowitz, Varshney, Wilkinson), while others question the importance of ethnic dynamics for conflict (Fearson & Laitin). Finally, works discuss ways in which governments can tame ethnic division and maintain political order (Lijphart).

Arend Lijphart, Democracy in Plural Societies: A Comparative Exploration (New Haven: Yale University Press, 1977), pp. 1-3; 16-52

Lijphart advances the theory that consociational forms of government can promote stable democracies in plural societies. In chapter 1, Lijphart askss how we can achieve stable democratic governments in plural societies? Previously people had argued that we needed to “homogenize” society and find political consensus. Lijphart argues that a society with homogenous preferences is not needed for a stable democracy–in fact the degree to which Western countries are homogenous is overstated. Another form of government can do the trick: Consociationalism! The defining characteristic of a consociational democracy is that diverse elites cooperate and that this counteracts the “centrifugal tendencies inherent in a plural society” (1). Consociationalism is both a normative model and an empirical model, as it helps explain how stability was maintained in several European democracies. In chapter two, Lijphart outlines four elements of a consociational democracy:

  1. A grand coalition including political leaders from all major groups in society. Grand coalitions are more conducive to parliamentary systems than presidential systems, but presidential systems can still have grand coalitions.
  2. Mutual veto. This ensures that a minority group is not always outvoted in a coalition. Lijphart is not super worried about “minority tyranny” where nothing gets done. If a group uses the veto too often, other groups will use it against the group as well. Also, the veto might not be used a lot, and simply acts as a deterrence against proposing things that would be really bad for a group. Finally, groups won’t want the government to be immobilized, and thus won’t overuse it.
  3. Proportionality for political representation. By this Lijphart means that groups should have proportional representation in government and that civil service appointments should be distributed proportionally.
  4. Segmental autonomy and federalism. I think this basically means there should be, to a degree, self-determination. Groups should be able to rule over themselves in areas where only members of their group are present, on issues where only their group members are concerned.
    Lijphart notes that consociationalism may lead to secession, perhaps by giving groups a taste of autonomy. Yet if a group feels so excluded from the state, even a more centralized government could probably not prevent secession. Lijphart argues, though, that on occasion secession may be desirable. Lijphart concludes this chapter by rebutting critiques of consociationalism. Some might say consociationalism is not very democratic because there is no strong opposition. Lijphart counters that it is the best possible option for plural societies. Some might also say that consociationalism does not promote liberty, equality, and fraternity, as it puts individuals into fairly rigid groups. Lijphart’s response is that consociationalism cares more about group equality than individual equality.

Robert Bates, “Modernization, Ethnic Competition, and the Rationality of Politics in Contemporary Africa,” in Donald Rothchild and Victor Olorunsola, eds., State Versus Ethnic Claims: African Policy Dilemmas (Boulder, CO: Westview Press, 1983), pp.153-167

Bates provides an instrumentalist and rational choice explanation for ethnic-based competition. Modernization theory suggests that ethnic conflict will decrease as a society modernizes. But we haven’t seen this happen, at least in Africa. Why? Bates takes an instrumentalist approach, arguing that ethnic conflict remains a rational way for coalitions to compete for scarce resources. The goods of modernity are scarce. Additionally, being more “modernized” than others makes it easier to obtain higher social rank. Thus in some sense, there is a zero-sum game. But why does it make sense for the coalitions that compete for the goods of modernization to form around ethnic lines? “Where modernization takes place often largely determines who gets modernized” (156). Thus to the extent that ethnicity overlaps with space, using ethnicity as a source of coalition-forming makes sense. Additionally, the “educated” often made more salient the “traditional” aspect of an ethnic group so as to mobilize a group to further his/her own political ambitions. But why do ethnic groups persist? Bates says they persist because they are successful at extracting goods for their members, and thus are able to maintain loyalty. The goods of modernity often result in prestige in the more “traditional” context.

Donald L. Horowitz, Ethnic Groups in Conflict (Berkeley: University of California Press, 1985), pp. 3-89

In this book, Horowitz provides a primordialist explanation for causes of ethnic conflict. What are the causes of ethnic conflict? What explains variation in the salience of ethnicity, both across time and place? Horowitz looks at divided societies in Asia, Africa, and the Caribbean. He looks at large groups that do not have hierarchical relations (e.g. he does not look at Hutu and Tutsi in Rwanda, he does not look at peripheral groups). Horowitz does not look at ethnic groups in the West because, inter alia, he argues that, “Ethnicity in the West typically does not displace all other forms of group difference” (19). Implicitly he is saying that ethnicity typically does displace all other forms of group difference in Asia, Africa, and the Caribbean. Horowitz admits that “separating out ethnic conflict in the West is a conceptual convenience, not a conceptual imperative” (20). There are ranked vs. unranked group, and this cleavage occurs if ethnic groups correlate with social class). Thus conflict in ranked systems has a class component, and there are fears of subordination. Conflict in unranked systems is characterized by issues of inclusion and exclusion. Neither system must be static. Ranked systems emerge from conquest, while unranked systems could emerge from invasion or migration. There are also large groups vs. small groups. For the former, ethnic conflict can occur at the center of government. For the latter, ethnic conflict is more likely to be peripheral. What is ethnicity? Horowitz resists a quotable definition. For him, ethnicity is an inclusive concept that includes race, language, and religion. Divisions may be defined at birth, but can change over time. Chapter two explores the nexus of ethnicity and family. Another way to classify ethnic groups: “Ethnic groups can be placed at various points along the birth-choice continuum” (55). And this classification can also apply to family. Under some conditions a person might be recognized as a relative, but under other conditions they may not be considered family. Horowitz notes that the language of intra-ethnic relations is often similar to language used to define family. Finally, ethnicity often builds on kinship, and the line between the two may be blurry.

David Laitin, “Hegemony and Religious Conflict,” in Peter B. Evans, Dietrich Rueschemeyer, and Theda Skocpol, eds., Bringing the State Back In (New York: Cambridge University Press, 1985), pp. 285-316

Laitin proposes that hegemonic powers can affect which cleavages do and do not become salient. Yoruba in Nigeria are about 40% Christian and 40% Muslim. Being Christian or Muslim is correlated with variation in socioeconomic opportunities. Christians have greater economic opportunities as a result of higher levels of education due to Christian missionary schools. So one might expect this to lead to a strong Muslim-Christian cleavage with political consequences. Yet among Yoruba religious identity does not affect political alignments. Rather one’s ancestral city affects political alignments. People are mobilized based on their ancestral city. Why? Why is religion not a salient political cleavage? Laitin argues that the hegemonic activities of the British colonizers explain the political salience of one’s ancestral city. Yoruba already saw traditional kings as legitimate authorities, but the British built on this, and empowered the kings. The British also actively worked to diffuse religious tension, fearing it would impair their ability to rule the Yoruba.

Barry Posen, “The Security Dilemma and Ethnic Conflict,” Survival 35, No. 1 (Spring 1993): 27-47

Posen asks: Why is there variation in intensity in post-Cold War Eurasian conflicts? The security dilemma, he argues, can help us understand this. Under what conditions will we observe or not observe a security dilemma? First, let’s remember what the security dilemma is. When there is imperial collapse, and some degree of anarchy, security becomes the number one priority of a state. If there is another group that wants power, both the group and the state will amass more weapons than are needed only for defense, or what the opposing party perceives as more weapons than are needed for defense. This will trigger the opposing party to amass more weapons, which triggers the other party to amass more weapons, etc. Posen’s definition of the security dilemma is: “What one does to enhance one’s own security causes reactions that, in the end, can make one less secure” (104). The security dilemma can be especially bad when offensive and defensive military technology look similar. Groups might use the military history of the opposing group to distinguish between offensive and defensive military buildup, but these estimations can be imperfect, The security dilemma can also be really bad when offensive operations are more effective than defensive operations. Opportunity is another aspect of the story. If one group (which could be the state) is in a stronger position than another group, and expects that its power vis a vis the other group might decrease over time, the group might want to use military action now. Or a state might want to take offensive military action against opposition when it sees that the international community is distracted with something, like perhaps the 2003 Iraq invasion. Posen looks at the case of Croat/Serb relations and Ukraine/Russia relations to test his theory. Of course the former have seen very bad inter-group violence, while the latter (at least when Posen was writing in 1993) had not. Posen argues that the former experienced a bad security dilemma. Incentives for preventive violence were great. At the end of the Cold War the two groups were not equal in power, and both groups had “terrifying oral history of each other’s behavior” (112). The risk of conflict between Russia and Ukraine was less great. For example, the two groups did not have terrifying oral histories about each other.

James Fearon and David Laitin, “Explaining Interethnic Cooperation,” American Political Science Review, 90, no. 4 (December 1996): 715-735

Fearon and Laitin aim to explain the norm of inter-ethnic cooperation. They argue that two equilibria can explain this norm: the spiral equilibrium and the in-group policing equilibrium. The puzzle: Interethnic relations are characterized by tension because interaction across groups is less frequent than interaction within a group. This means that there are less cross-group relationships, and that group A would have a tricky time identifying an opportunist in group B. So given this tension, why is inter-ethnic cooperation the norm? Why do we not see more inter-ethnic violence? The spiral equilibrium is one explanation. In this equilibrium inter-ethnic “cooperation is sustained by the expectation [that] if cooperation breaks down…members of each group will punish each other indiscriminately for some length of time” (730). This equilibrium predicts a long period of peace, and then if opportunistic behavior occurs, a long period of peace. The in-group policing equilibrium is another explanation. In this equilibrium, “people expect that someone who defects against an ethnic other will be identified and sanctioned by members of his or her own group” (730). This equilibrium predicts a long period of peace, and then if opportunistic behavior occurs, perhaps there will be a little violence, but peace soon will be restored.

Mark Beissinger, Nationalist Mobilization and the Collapse of the Soviet State (Cambridge: Cambridge University Press, 2002), pp. 1-40, 159-178.

Bessinger seeks to understand the collapse of the Soviet Union. 36 pages into this dense book, he mercifully offers his main point: “The basic argument of the book is that the disintegration of the Soviet state could not have taken place without the effects of tidal influences of one nationalism on another. Rather than simply being a manifestation of structurally predetermined conditions…the collapse of the Soviet state materialized out of a four-year period of ‘thickened’ history in which events acquired a sense of momentum, transformed the nature of political instutions, and assumed the characteristics of their own causal structure” (36). A bit more succinctly, nationalist movements among some peoples affected events among other peoples. He is arguing that “action can function as both cause and effect” (11). The diversity of responses of peoples to the Soviet nationalist messages can help us understand the role of structure and agency in manifestations of nationalism. He argues for the importance of studying events. Why? Events can be a consequence of the impact of structural forces, events can be an opportunity to look at a variety of forms of actions, events are a time when national identities are formed, and the outcomes of events affect identity. Pages 159-178 provide examples of how tides of nationalism interacted with structure and agency to affect new tides of nationalism among other peoples.

Ashutosh Varshney, Ethnic Conflict and Civic Life: Hindus and Muslims in India (Yale University Press, 2002), pp. 3-52

Varshney argues that the degree of inter-ethnic forms of engagement explains variation in inter-ethnic conflict. What explains variation in inter-ethnic violence? Previous approaches to understanding this question cannot account for inter- and even intra-city variation in Hindu-Muslim violent riots in India between 1950 and 1955. Varshney argues that the pre-existing degree of inter-group forms of engagement matters. He creates a dichotomy of inter-group forms of engagement: Associational forms of engagement and everyday forms of engagement. The former includes things like inter-group business groups and clubs, the latter things like going to school with non-co-ethnics. Both of these forms of engagement help manage inter-group tension and can prevent tension from becoming violent, but the former is stronger, and makes a place better able to peacefully withstand a “shock” (e.g., a politician attempting to mobilize anti-ethnic group sentiment). This is especially true in cities (where 96% of the riots Varshney looks at took place). In rural areas, everyday forms of engagement may have strong explanatory power, but in cities, where people are more anonymous, associational forms of engagement matter more. What are the mechanisms by which inter-group engagement reduces the potential for inter-group violence? Engagement helps kill rumors and promote inter-group communication. Varshney’s argument is that forms of engagement is a proximate causal variable. The actual causal chain goes something like this: Mass movements -> Civil order -> Violence or peace.

James Fearon and David Laitin, “Ethnicity, Insurgency, and Civil War.” American Political Science Review. 97(1), 2003, pp. 75-90

The conventional wisdom was (is?) that there was a sharp increase in the number of civil wars after the end of the Cold War. Fearon and Laitin argue that in fact there has been a steady increase in the number of civil wars since the period of decolonization, the 1950s and 1960s. The increase in the number of civil wars is due wars starting at a faster rate than wars ending. They also argue that ethnic or religious diversity and antagonism are not risk factors for civil war. (For any level of diversity, as GDP per capita increases there is a decreased chance of civil war.) Rather the conditions that favor insurgency–things like poverty, political instability, rough terrain, and large populations have explanatory power.

Ashutosh Varshney, “Nationalism, Ethnic Conflict, and Rationality” Perspectives on Politics, 1(1), 2003, pp. 85-99

To what degree can rational choice theories help us understand questions of nationalist or ethnic resistance? Often we see people engaged in ethnic-related resistance, even when the costs of doing so are very high. Traditional approaches to rational choice are ill equipped to explain this behavior. It can be helpful to distinguish between two types of rationality: value rationality and instrumental rationality. Value rationality emphasizes the utility one gets from behavior that is a function of values/beliefs, without regard to the likelihood of an action’’s success. Value rationality explains actions motivated by things like dignity and national liberation. Instrumental rationality is more in line with our traditional understanding of economic utility. It “entails a strict cost-benefit calculus with respect to goals” (86). Instrumental rationality can be helpful in explaining why, once ethnic resistance has hit some threshold level and has an increased likelihood of success, people would choose to whether or not to join a resistance movement. When these two approaches to rationality are combined, we can better understand the microfoundations of ethnic resistance, and ethnic mobilization. Varshney offers some examples of behavior that can be explained by these approaches to rationality. Pure value rationality: suicide bomber (though if we think about utility in the afterlife, this is also instrumentally rational). Pure instrumental rationality: when a group is close to gaining power, and you join it only so that you can reap some of the rewards of the group’s success. Combined value and instrumental rationality: when you participate in a movement both because of the utility you get from expressing your values and because of the gains from the possible success of the resistance.

Daniel Posner, “The Political Salience of Cultural Difference: Why Chewas and Tumbukas are Allies in Zambia and Adversaries in Malawi,” American Political Science Review 98, No. 4 (November 2004), pp. 529-545

Posner asks: When do cultural cleavages become politically salient? He looks at two ethnic groups, the Chewas and the Tumbukas in Zambia and Malawi, using as a natural experiment the fact that the boundary between these two countries was drawn with no respect for on-the-ground ethnic dynamics. The cultural differences between these two groups are identical in both countries. But in Zambia the groups get along just fine, while in Malawi they have a more antagonistic relationship. Posner argues that this puzzle can be explained by the size of the group vis a vis the size of the country’s political arena. In Zambia neither group is proportionally large (C: 7%, T: 4%), so ethnicity is not used as a basis of political mobilization by political entrepreneurs. In Malawi both groups are proportionally large (C: 28%, T: 12%), so political entrepreneurs mobilize each group by playing the ethnic card, and trying to instill Chewa fear of Tumbuka and vice versa. Posner supports his argument that both groups are similar culturally, but vary in how they see each other across countries through surveys. In total 180 people were interviewed in 4 villages–2 on each side of the border. Posner disregards other theories that could explain variation in ethnic antagonism. Modernization theory doesn’t have explanatory power here because both groups in both countries have similar levels of development. Variation in electoral institutions can’t help explain the puzzle because both countries use single-member plurality systems. Posner also finds variation in the social salience of ethnic cleavages. In Zambia there is greater willingness to marry across Chewa-Tumbuka ethnic lines than in Malawi. Posner ponders that perhaps the social salience of an ethnic cleavage could be a function of the political salience of an ethnic cleavage. What does all this mean for researchers? It is not necessarily the case that a cultural cleavage equates to a political cleavage. People who use the ELF metric, or similar measures, often inaccurately equate these two things.

Steven Wilkinson, Votes and Violence (Cambridge University Press), 2004, Chapter 1 (pp. 1-18)

Wilkinson argues that ethnic riots are often planned. Politicians might plan ethnic riots to “change the salience of ethnic issues…in order to build a winning political coalition” (1). Under what conditions will political competition lead to peace v. ethnic riots? Electoral incentives at the local level and level at which the government controls the police “interact to determine both where and when ethnic violence against minorities will occur, and more important, whether the state will choose to intervene to stop it” (1). The former can help explain where riots break out; the latter can explain when state governments intervene to stop the riots. Wilkinson finds support for his theory by looking at 2,000 Hindu-Muslim riots in India between 1950 and 1995.

James Habyarimana, Macartan Humphries, Daniel N. Posner, and Jeremy M. Weinstein, “Why Does Ethnic Diversity Undermine Public Goods Provision?” American Political Science Review 101, No. 4 (2007): 709-725

Previous work has linked ethnic diversity with low public good provision. This paper aims to identify the mechanism. The authors outline three types of possible mechanisms:

  1. Preferences: Co-ethnics have similar preferences; co-ethnics attach great utility to each others’ utility than to the utility of non-co-ethnics.
  2. Technology: It is easier to cooperate with co-ethnics because of shared language, culture, etc.; It is easier to find and punish co-ethnic non-cooperators.
  3. Strategy selection: There is a norm of cooperation based on expected punishment. (This is different from 2.b . Strategy selection is more like a “good” prisoner’s dilemma equilibrium.
    The authors tested which of these mechanisms was at play by conducting surveys and experiments on residents of a slum in the Ugandan capital. They find support for the technology mechanism. Specifically, co-ethnics are linked through a social network, and this increases credible threat of sanction, and thus increases cooperation. Perhaps the most well-known aspect of their experiment was a version of the dictator game, where respondents were given some money, and told to divide it up between themselves and two other players however they wished. Their identity and how much money they kept for themselves were kept secret. They were shown pictures of the two other players’ faces, which supposedly had clues about the other players’ ethnicities. This game tested “Preferences” hypotheses (above). The authors found no effect. Players on average gave the same amount to both co-ethnics and non-co-ethnics. One critique of this experiment is that perhaps people identify co-ethnics based on things besides one’s face. How one dresses and speaks, for example, might be more revealing.

Stathis N. Kalyvas, “Ethnic Defection in Civil War,” Comparative Political Studies, 41, No. 8 (August 2008): 1043-1068

Kalyvas looks at the implications for a constructivist perspective on the effects of civil war on civil war dynamics. Identities should be seen not just as a cause, but also a consequence of civil war. He argues that when war begins, new incentives appear that change the reasons that people choose to fight with one side or another. He looks at defection from ethnic-affiliated rebel groups as one example of the effects of civil war on the nature of inter-ethnic violence. In this process a member of one ethnic group joins an organization “explicitly opposed to the national aspirations of the ethnic group with which they identify” (1045). The individual’s ethnic identity, though, does not change. Another example of the effect of civil war is identity shift. An example of identity shift would be immigrants in the US identifying more and more with being American, resulting in assimilation. What explains variation in the level of ethnic defection? Kalyvas argues that this depends on a group’s willingness to recruit non-co-ethnics, which in turn depends on the group’s resources. Kalyvas looks at the German occupation of Greece in WWII, and tests the explanatory power of certain variables to explain why some Greeks fought with the much-despised Germans. He finds that areas that saw insurgent violence were more likely to have Greeks who defected to the Germans. Also, the more control the Germans had over an area, the more likely Greeks were to defect to the German side. Kalyvas finds that economic, social, and political variables have no explanatory power. The main point here is that we can learn things by thinking about civil war as an independent variable.

Voting, Elections, and Electoral Systems

The readings in this section focus on three main debates: 1) how are electoral system chosen, and in particular how did some countries come to select proportional representation (PR) systems? 2) What motivates people to participate in politics? And 3) How do electoral systems shape behaviors, and what are the consequences of specific electoral rules? During the era of limited suffrage in the 19th century, plurality and majority rule-based electoral systems were predominant. At the turn of the 20th century, most continental European countries moved to PR, while Anglo-Saxon countries kept plurality systems. During the last decades of the 20th century, mixed member proportional (MMP) representation systems have been adopted by some countries (e.g., Germany, Italy, Japan).

Carles Boix, “Setting the Rules of the Game: The Choice of Electoral Systems in Advanced Democracies,” American Political Science Review, 93, No. 3 (September 1999): 609-624

Boix enters in the debate about how electoral systems changes (which can be used as an example about the debate of how institutions in general change). The debate starts with Rokkan (1970) who interpreted the selection of PR as the result of a convergence of pressures from below and from above: the rising working class wanted to gain access to legislatures and the most threatened of the old, established parties demanded PR to protect its position against the new wave of voters created by the universal suffrage. Boix generalizes the Rokkan hypothesis:

  1. The consequences of electoral rule: these rules are constraining devices and induce strategic behavior; only a certain number of viable candidates compete, the number increase with PR. 2)
  2. The calculation of rulers and the stability of the electoral arena: anticipating that designers choose the system that maximize their chances of staying in power.
  3. The reform of the electoral system as a function of the viability of the old party system: the change to PR also depends on the interaction of two main conditions: the strength of the new entering parties (the socialist party at the turn of the century) and the coordinating capacity of the ruling parties (if they are tied in votes, coordination is hard). So if there is no new party or if there is a strong non-socialist party the change from majoritarian systems to PR systems will not happen. On the other hand, if the non-socialist side is very fragmented there will be strong incentives to change to a PR system.
    Boix also finds an interaction between country size and internal fragmentation appears important–in small-medium size countries, internal fragmentation favors PR.

Thomas Cusack, Torben Iversen, and David Soskice “Economic Interests and the Origins of Electoral Systems,” American Political Science Review 101, No. 3 (August 2007): 373-391

Cusack, Iversen, and Soskice introduce two additional theories to explain the adoption of PR systems. They bring in the idea of minimal winning coalition where PR systems promise redistribution and explain the rise of the welfare state through minimum winning coalitions and exclusion of the Right. The authors also bring in the idea of consociationalism where PR is a system that affords representation to all relevant groups as a means of solving policy disputes in a consensual manner. Cusack, Iversen, and Soskice argue that PR involves both exclusionary and inclusionary elements. Cusack, Iversen, and Soskice’s theory runs counter to that of Boix and also Rokkan—in particular against Rokkan’s idea that the key to PR adoption is the division among Right parties on non-economic issues (such as religion), which cannot be overcome to coordinate against rising Leftist working class. Cusack, Iversen, and Soskice argue instead that the Right adopted PR when its support for consensual regulatory frameworks, especially those related to labor market and skill formation where cospecifc investments were important, outweighted their opposition to the redistribution consequences. This occurred in countries with previously densely organized local economies. In the presence of cospecifc assets: workers can only use their skills in one particular industry which create higher incentives for both business and labor to cooperate (note: Iversen and Cusack are not new in considering the cospecifc assets as an important feature that play a roles in the development of different level of government spending. Implicit in their argument is that to attend to specialized interests, the political system must allow for the political representation of these interests. PR can provide this representation. In majoritarian systems, parties need to aim to the median voter and hence claim their independence from special interest groups. Historically, countries that adopted PR tended to have industrialization based on export specialization and specialization in area that required skilled workforce, small-scale industry in autonomous towns, history of guild activity, unions that developed as industry unions, confessional or regional but with cross-industry linkage, and coordinated employers.

Seymour Martin Lipset and Stein Rokkan, “Cleavage Structures, Party Systems, and Voter Alignments: An Introduction,” in Peter Mair, ed. The West European Party System (Oxford University Press, 2009), pp. 91-138

Lipset and Rokkan describe political parties as the “essential agencies of mobilization” (4). Parties have an expressive as well as an instrumental and representative function. They allow citizens to express their grievances and function as actors of political change. There are two dimensions of party cleavage: the territorial-cultural and the functional. Two revolutions, the national and the industrial, created four kinds of cleavages that can be located along these axes. The frst of these, the national revolution, created “the confict between the central nation-building culture and the increasing resistance of the ethnically, linguistically, or religiously distinct subject populations in the provinces.” It also created the church-state cleavage. The industrial revolution created the confict between landed interests and industrial entrepreneurs as well as that between owners and employers. Some examples are appropriate. The French Revolution created a bitter confict between the rising nation-state and the corporate claims of the churches (an example of church-state cleavage). This confict arose especially over the issue of education. The owner-employer cleavage led to the emergence of workers parties all over Europe. The openness of the system to social mobility determined to what extent the parties would be radical or “domesticated.” Having looked at the important cleavage structures, Lipset and Rokkan look at how they are translated into party systems. This does not happen automatically. Lipset and Rokkan distinguish between four such conditions, each representing a threshold for entry. The conditions are traditions for decision-making, channels for expression of protest, opportunities and costs of alliances, and finally possibilities and limitations of majority rule. The associated thresholds are: legitimation, incorporation, representationl, an dmajority power.

John Aldrich, “Rational Choice and Turnout,” American Journal of Political Science, 37, (1993): 246-78

Aldrich tries to explain changes in voter turnout through a rational choice model. However, he claims that rational choice account of voter turnout is different from a typical Olsonian collective action problem where high are costs and a small share of benefts lead to a free rider problem. Aldrish argues that voting represents a low cost and low expected beneft situation. Aldrich reviews three rational choice voting models. Basic Model: preferences determine political behavior. The cost of voting (including information collection, getting to the voting booth, etc.) is represented as C, and the cost of abstention is zero. When the cost of voting increase (e.g., new registration regulation), turnout should go down. This model also predicts that voters will abstain if costs are too high (C greater than 0.5) and voters will vote for the favorite candidate if C = 0. However, the model is has no prediction when 0 less than C less than 0.5. Calculus of Voting Model: this is based on the work of Downs (1957) and Riker and Ordershook (1968). These authors introduced the D term to represent “citizen duty,” or the influence on political participation and voting of the desire to see democracy continued. Also these authors add P, the probability assigned by each individual to the state of nature (how close the election is), and B, the beneft from your favorite candidate winning over the other candidate. The equation is R(reward) = P*B + D - C. The problem with this model is that the D term is hard to grasp, and P is hard to evaluate. Minmax Regret Model: Based on work by Ferejohn and Fiorina (1974), instead of maximizing utility as in Models 1 and 2, they minimize regret. The “regret” question is: “if it turns out that a given state of the world is true, would you have any regret that you chose the action that you did?” Which yields the maximum amount of regret? The maximum regret of abstention is to find out that your vote for your prefered candidate would have made a difference to the outcome. Minmax refers to those actions that would yield the minimum of all these maximum regrets. A problem with this models is that it predicts all voters will always vote for their first choice candiate, and will not engage in strategic voting. Game Theoretic Models: game theoretic accounts add the fact that people take into account what they think the others will do. The estimation of P in these models tend to be quite low, leading to low turnout prediction. As the size of the electorate increase these models converge to the results of Models 1 and 2. According to Aldrich, voting decisions are decision made at the margin, and hence, small changes in the cost and beneft structures can make a difference to turnout. Aldrich presents the Low Cost - Low Benefit Model where many variables are related to turn out, where citizens make errors in their decisions, where most of the action is decided by D and C, and where the strategic politician can manipulate D and C to change turnout. This model explains why close elections have higher turnout–politicians have information about P and invest accordingly. This model also explains the decline in turnout through D term, which Aldich does not believe to be politically inert. The D term is election specifc but also represents long term values and beliefs.

Kay Schlozman, Sidney Verba and Henry Brady, “Participation is Not a Paradox: The View From American Activists,” British Journal of Political Science, 25, (1995)

Through survey research, these author try to identify at an individual level where there is a paradox of political participation. The authors claim that political activism yields different types of gratifcation including many non-material types. From the perspective of rational choice theory, any of the selective rewards—whether material, social or civic–can function to provide benefts suffcient to justify the costs for a rational actor. There are three types of selective gratification: material gratification, social gratification, civic gratification, and one type of collective gratification, the collective outcome.

Bingham G. Powell and Guy Whitten. “A Cross-National Analysis of Economic Voting: Taking Account of the Political Context,” _American Journal of Political Science _, 37 (1993)

Powell and Whitten argue that in decision-making processes related voting for or against incumbents, economic issues matter but their relevant is filtered through other factors, such as consideration of the ideological position of the incumbent, its electoral base, the ease of assigning political responsibility for economic outcomes. Whether voters hold incumbents accountable for econmic outcomes depends on whether voters can assign responsibility for these outcome. Whether voters can assign responsbility depends on the level of voting cohesion of the majority party, the inclusiveness of the committee system in the legislature, whether there is bicameral opposition, whether the incumbent belongs to minority government, and whether there is a coalition government. Where clarity of responsibility is high, voters engage in economic voting–i.e., punishing incumbents for failing to make economic gains. As well, voters have different expectations of incumbents belonging to Left and Right parties, and as a result will have different responses to the performance of teh incumbent. Left-party incumbents are expected to do better with employment, and Right-party with inflation. Empirically, it seems that only Right governments are punished for failing to control infation.

Russell J. Dalton and Martin P. Wattenberg (eds.), Parties without Partisans: Political Change in Advanced Industrial Democracies (Oxford University Press, 2000), chapters 2 (Dalton) and 3 (Dalton, McAllister, and Wattenberg) [pp. 19-76]

In Chapter 2, Dalton identifies a trend of partisan dealignment in advanced democracies, which they attribute to changes in the level of education and the growth of mass media. Empirically, Denmark and Belgium display the lowest levels of dealignment, while Italy, Ireland and Austria the strongest. Dalton writes “Feeling of partisanship tap the popular vitality of representative democracy. Partisan ties bind individuals to their preferred political party, as well as the system of party democracy. Partisan ties also help orient the individual to the complexity of politics and provide a framework for assimilating political information, understanding political issues, and making political judgments.Partisan ties mobilize individuals to participate.” In Chapter 3, Dalton and co-authors provide empirical evidence of dealignment. At the individual level, there is an increasing tendency for voters to report that they have shifted their votes between elections, especially in nations where the decline in partisan attachment has been greater. There is evidence of “ticket splitting,” Finally, the timing of one’s voting decision has changed and more people state that they are making their decision during the campaign or even on the day of election. As partisanship has declined, the role of the candidate has increased and candidate-centred politics dominate. A symptom and cause of this has been the spread of the primaries. Similarly the spread of the television has freed candidates from reliance on the party. The authors also claim that this process is much stronger in presidential systems than in parliamentary ones. In fact, even if the trend appear visible in both types of systems, it is much more dramatic in the US and France than in Canada, Britain and Austria.

Herbert Kitschelt and Steven I. Wilkinson, “Citizen-Politician Linkages: An Introduction,” in Herbert Kitschelt and Steven I. Wilkinson, eds. Patrons, Clients, and Policies: Patterns of Democratic Accountability and Political Competition (Cambridge University Press, 2007), pp. 1-46

Kitschelt and Wilkinson critique the political participation literature for focusing too much on a “responsible party government” and ignoring patronage-base, party-voter linkages “the clientelistic accountability represents a transaction, the direct exchange of a citizen’s vote in return for direct payments or continuing access to employment, goods and services.” Despite of what modernization theories have predicted, this type of political relationship exists and is at the base of many elections (e.g., in Latin-America, Southeast Asia, etc.). Furthermore, this phenomenon has important implications for the economic growth of a country. Clientelism needs: 1) a cognitive condition, or the knowledge of the other side’s motivations and payoffs from alternative courses of action and 2) a motivational condition, or the voluntary and spontaneous compliance of constituencies with clientelistic inducements. Contingent direct exchange, predictability, and monitoring constitute clientelistic exchange. Contingent direct exchange is where nature of the goods provide defnitive evidence about the nature of the linkage. For instance, allocation of private goods signal clientelism. Public good, instead, are valence issues and cannot by defnition be traded in clientelistic exchange. Predictability and elasticity of citizens’ conduct means that the politicians needs to be confident that the target voter or group will vote in accordance with the exchange agreement. Finally, voter behavior is monitoring, the simplist method being the elimination of secret ballot. Parties make more effort to build principal-agent linkages of accountability whenever competitiveness is intense. This linkage becomes clientelism when there is a low level of economic development. Under conditions of democratic contestation, ethnocultural division may be a catalyst for clientelism; not necessarily because of the existence of strong ethnic markers but because of the existence of a strong network around a particular definition of ethnicity.

Kanchan Chandra, “Counting Heads: A Theory of Voter and Elite Behavior in Patronage Democracies,” in Herbert Kitschelt and Steven I. Wilkinson, eds. Patrons, Clients, and Policies: Patterns of Democratic Accountability and Political Competition (Cambridge University Press, 2007), pp. 84-109

Chandra argues that patronage and ethnic favoritism go together. We can explain ethnic favoritism as an outcome of the information constraints that characterize patronage transaction. In other words, when people use information short cuts to make decision, they will rely on ethnic markers as an heuristic. Patronage democracies are democracies in which the state has a relative monopoly over jobs and services and in which elected offcials enjoy signifcant discretion in the implementation of laws job allocation. Patronage democracy is a form of rent-seeking where the returns of the politicians are votes instead of bribes. In this setting, it is in the interests of the voters to coordinate to maximize gains from the vote selling. Similarly, the candidates have an incentive to target the distribution of individual benefts to group members rather than free foating individuals. Distributing the benefts to members of one groups sends a signal to the other members of the group that they can trust you. The superior visibility of ethnic identities in limited information environments also drives voters to obtain psychic benefts from co-ethnic rather than non-co-ethnics.

Arend Lijphart, 1999, Patterns of Democracy, Ch 8 (electoral systems) (pp. 143-170)

Disporportionality occurs when the proportion of votes is not commiserate with the proportion of seats allocated. Lijphart finds that plural electoral systems, small countries, presidental systems, and counties with fewer parties tend to have higher levels of disproportionality.

John M Carey and Matthew Soberg Shugart, “Incentives to Cultivate a Personal Vote: a Rank Ordering of Electoral Formulas,” Electoral Studies 14:4 (1995): 417-439

Carey and Shugart argue that seat allocation formulas affect candidates’ incentives to campaign on a personal basis rather than on party reputation. Variables that enhance personal vote-seeking are: 1) lack of party leadership control over access to and rank on ballots, 2) degree to which candidates are elected on individual votes independent of co-partisan, 3) whether voters cast a single intra-part vote or multiple votes or party-level vote, and 4) district magnitude, which affects incentives in opposite directions depending on variable (1). The authors find that personal reputation is least important under closed list, single round elections such as single member districts in the UK and closed-like PR in Israel. Personal reputation is most important single nontransferable vote, open endorsement systems.

Gary W. Cox, Making Votes Count: Strategic Coordination in the World’s Electoral Systems (Cambridge University Press, 1997), chapters 1-3 (3-68) and 10-12 (pp. 181-237)

Duverger law states that a majoritarian (or plurality-based) electoral system would lead to a two-party system at the district level. According to Satori, this district level bipartisanship will then translate in a two party system at the national level when parties nationalize. Cox addreses the question of why would politicians from different districts would ever link together to create a national party. Inter-district linkages occur when legislator are pursuing a national policy objective, when there is competition over presidency, when there is competition over premiership, when there are upper tier seats, and on issues of campaign finance. Furthermore, it’s important to consider the mechanisms through which the voters too behave strategically making consideration that go beyond their local interests toward national level issues. In fact, we could witness “strategic sequencing,” where voters consider the national viability of a candidate. “Strategic balancing,” for instance in the US when voters try to balance the president’s party in the mid-term elections; “threshold insurance,” when voters vote for an ally that risks not to pass the threshold. In Chapter 12 of the book, Cox examines whether there a trade-off between electoral systems that provide a stable governments (strong systems) and those that provide high representativeness? Representativeness is defined as the distance between the ideal policy position of the most poorly represented voters and the policy enacted by the government. In a two party system, minimizing this distance implies that the parties will converge toward the median voter. Therefore, in a Duvergerian world with two parties, a majoritarian system will lead to centrist policies that maximize representativeness. However, in a non-Duvergerian world with more than two parties, the outcome of the electoral systems depends on the level of coordination between voters and candidates. In a strong majoritarian system, if the center fails to coordinate extreme candidates, extremely candidates may win and pull the policy far from the median voters (e.g., Allende, Fujimori) and decrease representativeness. In contrast, more permissive systems as PR systems will be much less sensitive to the level of coordination between parties and could result in greater level of representativeness.

Political Economy of Advanced Democracies

Concept Map

While the term “Welfare State” has gained an extremely negative connotation in contemporary American politics, in the academic literature, the term refers to the study of the structure and the development of different systems of ensuring the living standards of citizens, including the sick, the elderly, and the unemployed. The study of the Political Economy of Advanced Democracies focuses on issues of welfare and redistribution for Western Europe and North America. Key questions that animate this literature include: what are the causes and implications of welfare state development? Is there welfare state retrenchment? And shapes patterns of redistribution?

Allan H. Meltzer, and Scott. F. Richard, “A Rational Theory of the Size of Government,” Journal of Political Economy 89 (1981), 914-17

Meltzer and Richard’s famous model of redistribution predicts that under universal suffrage and majority rule, a right-skewed income distribution implies that median income is below mean income, and the decisive median voter will vote for high taxes and redistribution. While Meltzer and Richard’s model is known as an explanation for why “more unequal democracies would vote for more redistribution,” the authors set out to explain “Wagner’s Law” of why governments get bigger over time. Assumptions: Budget is balanced. Individuals are utility-maximizers who are fully informed and not myopic. No public goods’redistribution and taxation are the only government activities. Hypothesis: when income distribution is skewed to the right, the median income falls to the left of mean income. Under majority rule, the median voter is decisive, and the median voter has an incentive to vote for redistribution of income (financed by taxes). As franchise is extended to include more voters below mean income, more people will vote for redistribution and the size of government (as measured by taxes) will increase. However, taxes and size of government will not increase infinitely because higher taxes and redistribution incur costs (reduced incentive to work) that lower earned income. Meltzer-Richard’s prediction of higher taxes in more unequal democracies contradicted empirically by the “Robinhood Paradox” of more redistribution in more equal democracies.

Gosta Esping-Andersen, The Three Worlds of Welfare Capitalism (Princeton University Press, 1990), Chapters 1-3 (pp. 1-78)

Esping-Anderson rejects structurally-based (industrialization) and institutionally-based (democratization) theories of welfare state development in favor of a balance of class power argument, in particular the power of the working class in creating the welfare state. The welfare state is defined by Esping-Andersen relative to its level of de-commodification (whether person can live without reliance on the market) and its system of stratification (how it orders social relations). Three types developed depend on union structure, the existence of working class-farmer alliance (existed w/ rural economy was capital intensive), and whether or not and how the middle was incorporated. Liberal (“social assistance”, “means tested”): social rights depend on demonstrated need; minimal de-commodification; social dualism between poor recipients and others; middle-class not wooed to welfare state. (US, Canada). Conservative (“corporatist”, “Bismarckian”: compulsory state social insurance, benefit depend on contribution of employer, intervene only when family’s capacity fails; limited de-commodification; preserve existing class/status groups; middle class occupationally segregated; labor-intensive agricultural allied w/ Catholics (Germany, France, Italy). Social Democratic (“flat-rate”): universalistic benefits at levels of the middle class with full employment guarantee; high de-commodification; worker-farmer alliance. (Scandanavia).

Geoffrey Garrett and Peter Lange, “Political Responses to Interdependence: What’s ‘left’ for the Left?” International Organization 45, No. 4 (Autumn): 539-564

The traditional view is that with increasing financial integration, countries, and partisan leaders, will limited in the economic strategies they can adopt. However, Garret and Lange argue that there are multiple paths to strong aggregate economic performance, and that corporatist and weak-labor countries adopt different strategies to pursue partisan goals. For the authors, countries like Austria, Norway, and Sweden fall under the “corporatist” lable while Canada, France, Japan, and the US fall under the “weak-labor” label. In addition to traditional politics such as welfare and full-employment, corporatist countries with left-governments pursue active labor market policies (e.g., job re-training) to adjust the workforce while maintaining full employment and pursue investments in the national economy with government subsidies, public investment, and corporate tax benefits. Weak-labor countries right governments typically have lower levels of taxation, but do not completely abstain from intervention in the market. Even during stagflation, right government protected business with bailouts and subsidies. While Garret and Lange find evidence of partisan separation with respect to government spending and taxation, there is no systematic partisan separation with respect to fiscal and monetary policies.

Paul Pierson, “The New Politics of the Welfare State”, World Politics (1996): 143-179.

Pierson argues that structural, class coalitional, and institutional theories of welfare state development cannot adequately account for welfare state retrenchment. Pierson argues that retrenchment is characterized by blame avoidance and the lack of radical change because the welfare state is itself an institution, which has created new interest groups that make it difficult for governments seeking re-election to radically alter the existing structure of the welfare state. Retrenchment includes increase in means-tested benefits, transfer for responsibility to the private sector, and changes in eligibility rules. Examining the US, UK, Germany, and Sweden, Pierson finds little evidence of fundamental changes to each country’s welfare state structure; however, his cases suggest that welfare retrenchment is facilitated when a government has significant political power (Thatcher and Reagan early in tenure), when the government is faced with budgetary crisis (Sweden), and when the state’s power is decentralized so that retrenchment advocates can shift the blame for program cuts (US). In Pierson analysis, he suggests that organized labor, which was instrumental to the development of the welfare state, is of declining importance as the welfare state has created new interest group and constituencies.

Evelyne Huber and John D. Stephens, Development and Crisis of the Welfare State (University of Chicago Press, 2001), pp. 14-32

Huber and Stephens seek to explain variation between welfare states (including expansion and retrenchment) using a framework of class and gender mediated by political parties. According to their theory, characteristics of different welfare states are influenced by the long-term pattern of partisan (left-labor or right-capital) governments. Long-term patterns of partisanship do so by changing the preferences of actors, universe of actors, and expectation of actors in four ways: 1) a ratchet effect where policies condition preferences (similar to Pierson), 2) structural limitations on policies, 3) legacy of production regimes linked to welfare states, 4) ideological hegemony of the labor movement. The organization / centralization of unions, the incumbency of left parties, and the opportunities available to capital all contribute to the ability of labor movements to promote social consciousness, which in turn influences their electoral success, which then leads to welfare state differences. Where there is greater dominance of social democratic ideology within labor movement and social consciousness among wage earners, the power of labor over capital is greater. Where there is high female labor force participation, strong women’s organization, and political allies available for women’s organizations, there is greater social consciousness among women. There are 5 classes: bourgeoisie (owners of capital/labor), petty bourgeoisie (small firms owners), upper-middle (professionals/managers), lower-middle (nonmanual employees), working class, and farmers.

Peter A. Hall and David Soskice, Varieties of Capitalism (Oxford, Oxford University Press, 2001)

Hall and Soskice take a firm-centered approach to create a framework for comparing developed political economies. Firms must solve coordination problems in industrial relations (wages/working conditions), vocational training (workforce skills), corporate governance (accessing finance), inter-firm relations (with suppliers and clients), and employee cooperation. History creates formal and informal institutions that provide for the relationships firms develop to resolve these coordination problems. There are two types of economies: coordinated market economies and liberal market economies. Coordinated market economies (Germany) coordinate with non-market relationship and use relational contracting. There is industry-level bargaining between unions and employers to equalize wages, vocational training to give workers industry and firm-specific skills, private and inside information in accessing capital, technology transfers through cultivated relationships, and limitations on unilateral action by top managers. Innovation is typically incremental. Liberal market economies (US) coordinate via hierarchical competitive market arrangements and formal contracting. There is a focus on general skills, access to capital based on public information, technology transfer through employee mobility, and top managers have unilateral control. Innovation is radical. Social policy typically matches the type of economy (e.g, liberal market economy with liberal welfare state). Globalization unlikely to lead to a common model.

Harold Wilensky, Rich Democracies (University of California Press, 2002), pp. 3-14

Wilensky takes the modernization approach to comparative economic development’that as rich countries get richer, they development similar economic, political, and social structures. Industrialization is the widespread use of tools that multiply effect of initial energy application and use of inanimate sources of energy. Industrialization is continuous, but the threshold for “fully modern” is reached when 75% of the labor force is not in agriculture and when 40-50% of women are working in nondomestic setting. Sounding much like Inkeles writing nearly 40 years before, nine structural and demographic changes (e.g., changes in kinship, more higher education) occur in these “fully modern” advanced industrial, rich, democracies. Among these structural changes is the emergence and growth of the welfare state. These structural and demographic changes imply increased mobility aspirations for parents, reduced economic value of children, reduced incentives to care for aging parents. Most prominently, as shown in figure 1, there is increased (non-agricultural) labor force participation by women, lower birth rates, and increased divorce rates, which necessitate certain governmental responses. Although Wilensky does not spell this out in the 10 pages here, in essence, modernization means that the state rather than the family becomes the primary provider of welfare goods.

Torben Iversen and David Soskice, “Electoral Institutions and the Politics of Coalitions: Why Some Democracies Redistribute More Than Others,” APSR 100:2 (2006):165-181

Iversen and Soskice argue that electoral institutions help explain the Robinhood Paradox (see Meltzer-Richard notes). After institutional variables taken into account, more inequality mean more redistribution; but without institutional variables, more inequality means less redistribution (see Table 5). Specifically, Iversen and Soskice’s argument is that PR systems facilitate left-center coalitions that favor redistribution while in Majoritarian systems, middle income voters are more likely to favor the center-right party, which does not favor redistribution. Unlike the Meltzer-Richard model, Iversen and Soskice assume that taxes and transfers differ across income groups (L=low, M=middle, H=high income). As a result, middle-income voters fear taxation by low-income people, but middle-income people also have an incentive to ally with low-income people to tax high-income people. In a PR system, there are three parties representing L, M, and H interests, so middle-income people can vote directly for the M party (parties are bolded), and then the M party forms a coalition with the L party to tax the rich. In Majoritarian system, there are only two parties’center-left (LM) and center-right (MH), and if middle-income people vote for LM, LM party may take up the L position and tax both middle- and high-income people. Therefore, because of the limits of representation in a Majoritarian system and out of fear of being taxed, M votes for MH. This is the rationale for why center-left coalitions emerge in PR and center-right in Majoritarian systems.

Jacob Hacker, “Privatizing Risk without Privatizing the Welfare State,” American Political Science Review 98, No. 4 (2004): 243-260

Hacker offers a counter argument to the prevailing view (in Pierson) that welfare state retrenchment has been largely incremental. Hacker shifts our perspective from the formal rules of welfare policies to the social consequences of these policies, in particular to who bears risk. Risk socialization occurs when risks are spread across citizens. Risk privatization occurs when risks are left to individuals and families. Hacker argues that significant retrenchment has occur in the US as risk privatization has occurred through “subterranean” and “stealth” tactics. Hacker argues that risks are increasingly privatized as a result of policy drift and conversion. Drift refers to changes in the operations and effects of policies that occurs without changes in policy structure. Drift happens because new risks have occurred in a changing social context (e.g., increased earning inequality, increasing population aging from decreased birth rates), and policy does not catch up to cover these new risks. Conversion occurs when the function and role of existing institutions are changed. Conversion and drift (and retrenchment) have occurred in health care where formal policies have not changed but outcomes have, while in social security formal policies and outcomes have remained stable (no retrenchment).

Philip Manow, “Electoral Rules, Class Coalitions, and Welfare State Regimes, or how to Explain Esping-Andersen with Stein Rokkan” Socio-Economic Review (2009) 7: 101- 121.

Manow supports Iversen and Soskice’s argument of the effect of electoral institutions on redistribution but seeks to disaggregate the higher level of redistribution seen under the PR system into two types of Esping-Andersen’s world of welfare’social democratic welfare states (in Scandinavia) and conservative welfare state (in continental Europe). Manow argues that Nordic countries have “red-green” coalitions under PR while continental Europe has “red-black” coalitions under PR. “Red” always refers to the labor movement. “Green” refers to agricultural parties. “Black” refers to Christian democratic parties. Manow uses Lipset and Rokkan’s political cleavage framework to explain why there were “Green” parties in Scandinavia and why there were “Black” parties in continental Europe. Nordic countries were homogenous Protestant states where there was no state-church conflict. Instead urban-rural socioeconomic divides existed during democratization such that agrarian interests came to be represented by “Green” agrarian parties. In contrast, continental Europe was characterized by church-state conflict (conflict in Lipset and Rokkan’ss 2nd critical junction) such that church-state conflicts were “particized” through “Black” Christian Democratic parties.